Anda di halaman 1dari 68

FOREIGN SERVICE

OFFICER TEST
5TH EDITION

STUDY GUIDE

Study Guide for the Foreign Service Officer Test


Introduction ........................................................................................................1 Purpose and Use of the Study Guide ....................................................1 What is the FSOT? ...............................................................................1 Test Dates .............................................................................................2 Disability Accommodations .................................................................2 Preparing for the FSOT.....................................................................................3 FSO Selection Process..........................................................................3 Step 1: The Registration Process ..........................................................3 On the Day of the Test .............................................................5 Test-Taking Strategies ..............................................................5 Procedures at the Test Center ..................................................6 Computer-Based Testing Procedures .......................................7 Step 2: Taking the FSOT ......................................................................7 Knowledge and Skill Areas on FSOT ......................................9 Suggested Courses of Study................................................... 11 Once the Test Session Is Over ............................................................12 Step 3: Qualifications Evaluation Panel Review Process...................13 Step 4: Invitation to Take the Oral Assessment ..................................14 Job Knowledge Test Section ............................................................................15 Sample Questions for the Job Knowledge Test ..................................15 Rationales for Sample Job Knowledge Questions..............................21 Biographic Questionnaire Test Section ..........................................................35 Sample Items for the Biographic Questionnaire ................................35 English Expression Test Section......................................................................38 Sample Questions for the English Expression Test ............................38 Rationales for Sample English Expression Questions........................51 Sample Written Essay Topics ..........................................................................60 Suggested Study Materials ..............................................................................61 The Oral Assessment ........................................................................................63

Every attempt has been made to ensure the accuracy of this publication. If, however, you should find any discrepancy between the information here and the information on careers.state.gov, please assume that the website has the most accurate and up-to-date information. If you have any questions or comments about this Study Guide, please write to:
FSOT Study Guide ACT, Inc. Workforce Development Division 101 ACT Drive P.O. Box 168 Iowa City, IA 52243-0168
2012 by ACT, Inc. All rights reserved. P.O. Box 168, Iowa City, Iowa 52243-0168 No part of this publication may be copied, reproduced or distributed in any form, including without limitation through electronic networking, distribution or retrieval systems, without the prior written permission of ACT, Inc.

INTRODUCTION
Purpose and Use of the Study Guide
The main purpose of this Study Guide is to explain the selection process for becoming a Foreign Service Officer and to prepare candidates to take the Foreign Service Officer Test (FSOT), one of the important steps in the selection process. This introduction presents a brief overview of the FSOT, its components, and the testing process. It also gives candidates a preliminary glance at the contents of the Guide. Real-life stories from Foreign Service Officers are highlighted in the left column throughout the Guide. The next section titled Preparing for the FSOT discusses the four steps in the selection process and the qualifications that must be met to become a Foreign Service Officer. This section provides detailed instructions and procedures to help candidates prepare for taking the FSOT. It also explains the knowledge and skill areas covered on the test, and it offers a list of suggested academic courses for study. The Sample Question sections in the Guide are designed to familiarize candidates with the types of items that will appear on the FSOT. Candidates are informed about the content, length, format, and time limits for each section of the test.

FSO STORIES FROM THE FIELD


I was a newly assigned Political Military officer in a Scandinavian country. As an African-American woman on her second day in the country, I attended a meeting at the Ministry of Defense regarding NATO issues. I arrived at the meeting hall where people were milling around. As I didnt know anyone there (yet), I sat down and waited for the meeting to proceed. After five minutes with the Chair looking at his watch repeatedly, I asked the person next to me what we were waiting for. He glanced at me briefly and said, We are waiting for the American he hasnt arrived yet. I said nothing. After a few more minutes went by, the Chair rose and stated, Well I guess the American isnt coming today. At this point, I gave him a smile and a wave. First, he looked puzzled, then shocked, and finally he smiled hurriedly so as to cover up his initial two reactions. Much later, when I got to know the Chair better, he sheepishly admitted to not having expected the American diplomat to be either female or black. He went on to say, You, Americans. We never know what to expect when an American is coming through the door for the first time. I have repeatedly heard this sentiment while overseas in some form or fashion and it makes me proud to have been able to serve the United States. Female Political Officer, 26 years experience

What is the FSOT?


The FSOT is a computer-based exam that consists of four separate test sections: 1. 2. 3. 4. Job Knowledge Test Biographic Questionnaire English Expression Test Written Essay Test

The first three sections of the exam contain items in a multiple-choice format. The Written Essay appears in the last section of the FSOT. Each section of the exam is timed separately and must be completed within the designated time limit.

FSO STORIES FROM THE FIELD


As an FSO, you seldom realize the true impact of your actions. As the Consular Section Chief in Calcutta, it was near the end of the day when the consulate received word that two missionary children had been admitted to a local hospital. Along with a member of our American Citizens Services staff, I immediately proceeded there and was escorted to the familys room. The father and mother were in their 30s. They explained that they had been in India for six months and their mission was some 80 miles north of Calcutta. Their two daughters, ages four and six, had become seriously ill and were rushed to the hospital in Calcutta where the physicians determined they were exposed to E. Coli poisoning. The youngest daughter recovered, but the eldest daughter remained in intensive care. As we entered the intensive care unit, it was clean, quiet, and dimly lit. The child, unconscious, was attached to several monitors. She was not doing well, and the prognosis was not good. I recall the mother lovingly stroking the childs black hair. I spoke with the attending physician and staff and gave them my card. As we left, I informed the family they should call us for any assistance they might need. I felt helpless; there was nothing I could do. Two years passed, and I was in my office in Washington, DC, providing a briefing to an FSO who was scheduled to begin work at the Consular Section in Bolivia. When he learned I had been stationed in India, he related a story that his wife a former missionary had told him. She had been assigned to India and had heard about two seriously ill missionary children being sent to a hospital in Calcutta. While the younger child soon recovered, her older sister remained in critical condition. Anguished, the family felt that the hospital staff was not doing enough to help her. The family related that the American Consul from Calcutta visited them and also spoke with the hospital staff. They added that following the Consuls visit, their child received increased care and attention and she survived. I felt I had done nothing, but that young couple credited me with helping save their childs life. Male Consular Officer, 14 years experience

The Guide provides sample questions for each component of the FSOT, but with a special focus given to the Job Knowledge and English Expression test sections. For these two sections, sample questions are provided along with their correct answers and rationales. The rationales explain the knowledge area tested by each question, why the correct answer is the best response, and why the alternate choices are incorrect. Although the Written Essay section is an important aspect of the test that is used to determine a candidates qualifications, it will not be scored unless the candidate passes the Job Knowledge, Biographic Questionnaire, and English Expression sections of the test.

Test Dates
The FSOT is administered three times per year with an 8-day testing window for each administration. Although the FSOT is given three times annually, candidates are allowed to take the exam only once a year. The exam is offered throughout the United States and at many U.S. Embassies and Consulates overseas. Candidates can visit the Department of States website (careers.state.gov) for the schedule of upcoming test dates.

Disability Accommodations
In addition to preparing and administering the FSOT, ACT, Inc is responsible for all special accommodation requests for candidates with disabilities. Candidates who need accommodations to take the test are requested to register at least 4 weeks before the next testing window. Accommodation requests must be submitted in writing with accompanying documentation at least 4 weeks before a requested test date for ACT to have sufficient time to make the necessary arrangements. Candidates who plan to request accommodations should carefully check their support documentation beforehand. NOTE: Documentation more than 5 years old will not be accepted.

PREPARING FOR THE FSOT


The FSOT is only one of the four important steps in the selection process for becoming a Foreign Service Officer. The four steps involved in the selection process are as follows:

FSO STORIES FROM THE FIELD


In the early '90s, I was assigned to the Regional Security Office in San Salvador. The Civil War was grinding to a close, with the Communist FMLN guerillas agreeing to stay within specific Zones of Concentration; these were essentially reservations far out in the hinterlands. As part of the peace process, the Charg decided to visit one of these Zones, located north of the Capitol. With one Salvadoran driver/escort a member of the bodyguard detachment I was sent overland to do an advance on the site. Before leaving, I was cautioned to ensure that the FMLN knew that the Charg would be flying in by helicopter the next day, as this group had recently fired upon a light plane that overflew their area. Upon arriving in the heavily patrolled Zone, I soon realized that I was the only English speaker for miles around. In discussions with the Commandante, he frowned when I told him that the Embajador would be arriving by helicopter. Is he coming in a green military helicopter? he asked in Spanish. Oh no, I assured him. He would be coming in a yellow civilian helicopter. That is good, he said, telling me something about helicopters in rapid Spanish. Depending upon the verb-ending, he was either saying: We used to shoot at helicopters, We do shoot at helicopters, or Were going to shoot at helicopters. Cursing myself for not having paid better attention in my Spanish classes at the Foreign Service Institute, I tried to clarify, saying But tomorrow we are not shooting at helicopters? No, he laughed. No problem. All the muchachos would be told. Fortunately he was right, and the Charg and his wife landed unopposed the next day.
Male Diplomatic Security Officer, 25 years experience

Step 1: The Registration Process


ALERT: Candidates should thoroughly read the Guide to the Foreign Service Officer Selection Process on careers.state.gov before they submit their registration. Candidates should complete their online registration form with care (see the Guide to the Foreign Service Officer Selection Process above). The registration form is more than just a sign-up sheet to take the computerbased FSOT it is the first form the Qualifications Evaluation Panel (QEP) will consider when reviewing the files of candidates who pass the FSOT. It is also the one document, along with the candidates Statements of Interest, that assessors will review before the Structured Interview on the day of the Oral Assessment. Once candidates submit their registration forms to ACT, they will not be able to change any of the information, except for contact information (addresses and e-mails), so they should fill out the registration form with the assumption that a Department of State official will be reading it.

FSO STORIES FROM THE FIELD


When South Africa held its first multi-racial election, I volunteered to be an election monitor. The election was held under the auspices of the Independent Electoral Commission, or IEC. I was asked to monitor three polling stations in downtown Johannesburg while several of my Consulate colleagues covered the suburbs and outlying districts. My first priority was to examine the main polling station located in Johannesburgs City Hall. On the morning of the election, I arrived at the Consulate very early and began my six-block walk to City Hall. My IEC cap was too small so I carried it along with my IEC armband. Although the turnout for the election was expected to be huge, I had no idea that a sea of first-time voters would extend more than three blocks from the polling station at 7:00 A.M. As I tried to think of a way to get to City Hall, I became aware that my presence was making many of the expectant voters, at the end of the line, uncomfortable. Holding up my cap and armband, I yelled IEC. The effect on the crowd was electric. Many of these would-be voters were understandably skeptical that free and fair elections could be held in a nation that had not yet totally shed all aspects of Apartheid. As others took up the chant of IEC, the crowd realized that the election would indeed be monitored. As if by magic, the crowd parted and for three city blocks I saw thousands of exultant, cheering faces, most of which were wet with tears of joy. To this day, I believe that my feet never touched the ground until I arrived at the front steps of City Hall. Of the many uniquely FSO experiences Ive had, this opportunity to be a part of such an epochal event was very special. Male Management Officer, 25 years experience

Choosing a career track during the registration process is very important. Candidates should take care to make the correct selection of their career track. Once a registration is submitted, candidates are locked into that career track for the duration of their registration candidacy. No changes in the selection of a career track are permitted, unless a year has passed and the candidate is able to submit a different registration, virtually starting the whole process again from scratch. Candidates should list all of their jobs whether paid or unpaid, part time or full time, and any continuing or significant volunteer activities. No jobs should be discounted. Being a waitress, assembling sandwiches at a food court, dog walking, or volunteering at a food pantry can all demonstrate hard work, persistence, customer service, and other skills of value to employers, including the Foreign Service. The Special Accomplishments box should be used to highlight skills that might make a candidate stand out from the crowd, whether it be Eagle Scout status or knowledge of American Sign Language. Type information directly into the online registration form, rather than cut and paste it from another source. Conflicts in the underlying coding of different word processing programs oftentimes do not manifest themselves until the data is further processed by a subsequent computer system that is, what looks fine to a candidate at the beginning of the process may look like gibberish to ACT and the Department of State later on. Candidates should print their registration forms before hitting the submit button. The registration forms become unavailable to candidates once they are submitted. Thus, it will not be possible to refer to them, edit them, or add information. Candidates should make a hard copy of their registration form to refer to later in the process. Carefully proofread the registration form and other materials before submitting them. When providing a contact e-mail address during the registration process, it is best to avoid using school e-mail addresses, which are often decommissioned when candidates graduate. Completing the registration form will take much longer than expected. Candidates should not wait until the last hour before the deadline to submit it. Servers get clogged, computers crash, and IT support is not available at midnight.

FSO STORIES FROM THE FIELD


As the holder of the fisheries portfolio at Embassy Managua, I was required to accompany U.S. fisheries officials to Nicaraguas Miskito Coast on an annual basis to ensure that shrimp trawlers were outfitted with Turtle Excluder Devices (TEDs) that would prevent endangered sea turtles from becoming entangled in the nets and drowning. The residents of the Miskito Coast were separated from the rest of Nicaragua by geography (an impenetrable rain forest with no roads between the Pacific and Atlantic coasts), heritage (they were descendants of pirates, shipwrecked slaves, and various Indian tribes), and language (they spoke English, Miskito Creole, or various Indian languages rather than Spanish). It was possibly because of this sense of being different that they were sensitive to being treated with respect, and our Deputy Chief of Mission reminded Embassy personnel to make sure we dressed professionally when traveling to the area, even though it felt like going on safari to us. I tried to strike a balance between professionalism and practicality the first time I traveled to check for TEDs. Instead of a suit, I wore a cotton shirt, a lower-than-my-knees jean skirt, and rubber-soled topsiders. I maneuvered around the first few ships with ease. Then the team boarded a small boat and motored out to a shrimp trawler farther out on the water. Boarding a boat from a dock is a very different thing than boarding a ship at high seas. One glance showed me I wouldnt be able to clamber the 20 feet up the rope ladder to get to deck unless I hiked my skirt up to my hips, so I told my team to go on without me. However, one of the crewmen a smelly, gap-toothed fisherman dressed in filthy clothes had other ideas. Without saying a word, he picked me up, threw me over his shoulder like a sack of flour and clambered up the side of the ship, no joke. It probably only took ten seconds, but it seemed like hours all under the gaze of about two dozen men including sailors, Nicaraguan government fisheries officials, and members of my own U.S. delegation. And then I had to get back down to the boat the same way. Truly, I wanted to die. They didnt prepare us for this in A-100 class. Female Economics Officer, 23 years experience
5

On the Day of the Test


The FSOT is given three times a year throughout the United States and at many U.S. Embassies and Consulates overseas. Although the FSOT is offered three times annually, candidates are allowed to take the exam only once a year (typically in February, June, or late September/early October). Thus, candidates should ensure they are ready to do their best on the test to avoid unintentionally setting their candidacies back a year.

Test-Taking Strategies
Be sure to get plenty of rest before taking the test. Candidates who are tired cannot possibly perform at the best of their abilities on the test. The day of the test is also not the time to start a new diet or exercise routine, either of which could have unintended consequences on a candidates performance. Candidates should maintain a normal regimen in the days leading up to the test. Know where the test center is located. Candidates should familiarize themselves with how to get to their test center by checking online maps or visiting the test center beforehand. Make allowances for stalled rush-hour traffic, broken-down metro trains, no-vacancy parking lots, and taxi drivers who cant find the test center. Dress comfortably. Taking the FSOT is not a job interview, and candidates will not come in contact with the Foreign Service Officers who may be interviewing them later in the process. Wear clothes appropriate to sitting at a computer console for 2-3 hours. Keep in mind that candidates are required to remove all suit coats, sweaters, and jackets, etc. Thus, candidates who arrive at the test center wearing a halter top under a jacket may find themselves cold and uncomfortable during the testing period. Avoid wearing strong fragrances, noisy jewelry, or other distracting attire. No sunglasses or hats are permitted. No food or drinks (including coffee) are allowed in the testing room. Candidates should have sufficient nourishment before the testing period. Candidates with previously approved special accommodations requiring the provision of food or drinks (for example, a candidate with diabetes or hypoglycemia) can request access to food or drinks (to be provided by the candidate) through the special accommodations process.

FSO STORIES FROM THE FIELD


I was posted in Vietnam during the Y2K scare when officials feared that computers worldwide would fail as the world passed from 1999 to 2000. In response to cable inquiries, I sent several reports back to Washington detailing what would be at risk in Vietnam should all the computers crash with the New Year (short answer nothing). As a reward for my assessment, I was appointed to be control officer for a team from the Inspector Generals Office who did not believe a country could be so uncomputerized as to have almost no risk from the year 2000. The second day of their visit, I took them to my favorite Hanoi restaurant and, as we were ordering, the power went off. The visitors assumed we would have to leave. I assured them that the restaurants in Hanoi dealt with these twoto four-hour blackouts nearly every day and simply cooked outdoors over charcoal. Indeed, the only concession to the lack of electricity was to open the windows, and within a half-hour our delicious, hot meals were served. The team started to believe me that the country was still "analog," and left the country feeling confident that Y2K would not cause problems. I never told them the charcoal used to cook their food was made (by hand) from cow dung in a village an hour away.
Female Management Officer, 31 years experience

Procedures at the Test Center


Candidates should plan to arrive about 15 minutes early to check-in at the testing center. All candidates will need to provide a current, government-issued ID. Expired IDs or school IDs are not accepted. Candidates without a current, government-issued ID will not be allowed to test. Candidates who registered in one name but whose legal name is different on the test day (for example, female candidates who marry and legally change their names) should contact ACT prior to taking the FSOT. Candidates who register in one name but present a government-issued ID in a different name will not be allowed to test unless they also bring legal documentation explaining the name change. All candidates will be required to have their photos taken at the test center for security purposes. All candidates will be required to sign an Examinee Agreement and Sign-In (EASI) form. Turn off cell phones and other electronic devices do not just put them on vibrate. Candidates will not be permitted to bring their cell phones or other electronic devices into the testing room. There will be no access to cell phones during the approximately 3-hour testing period. Work colleagues and family members should be informed of this situation beforehand, and any necessary arrangements made. Leave valuables at home, especially electronics. While some test centers have individual lockers available, many do not. In case there are no lockers, coats and purses should be left at the front of the testing room. Candidates will not be allowed to leave the immediate area during the testing period. Stepping outside to get a breath of fresh air while taking a bathroom break constitutes leaving the test center, and that individuals candidacy will be terminated. Candidates should write down their ACT passwords and user names to facilitate the computer check-in process. While some testing centers will be able to sign-in candidates electronically, others will not. There is no way to predict which system will be available in the test centers beforehand. Some note-taking material either scratch paper or a dry erase board will be provided to candidates. Candidates are prohibited from bringing in their own scratch paper or writing utensils.

FSO STORIES FROM THE FIELD


On the whole, I firmly believe that Foreign Service life has been a plus for my family. My children one born in Singapore, the other in Ecuador have developed a confidence and adaptive skills I never could have imagined in my childhood. But there were days and nights when my wife and I questioned our sanity. One such night was early in our first-borns life, back in 1993. Assigned to Bogota during a particularly violent period, we had rationalized that a toddler would be safe, as he certainly wasnt going anywhere outside of our arms and his stroller. We were all a bit jumpy as the drug war violence had hit close to home, with an explosion at a pizza restaurant a block from our elegant Bogota apartment. It was shortly after wed put our pride-and-joy to bed when the rat-a-tat-tat sounds exploded right outside the nursery window. My wife and I, in tandem, sprinted into the babys room, pulled him from the crib, and rolled under the crib, crouching together under the window from where the sounds seemed to emanate. It seemed like an hour but was probably only a minute before we realized that the explosions were not the final showdown for Pablo Escobar, but yet another Colombia birthday party punctuated with fireworks and, soon to follow, mariachis. We slowly very slowly stood up and danced a little dance to celebrate the gift of life.
Male Public Diplomacy Officer, 23 years experience

Individual candidates begin their exams as soon as they are completely checked in. Candidates who arrive later should attempt to be quiet when entering the testing room to avoid disturbing those already testing. Ear plugs are normally available at domestic test centers.

Computer-Based Testing Procedures


The FSOT is a computer-based test, and technology sometimes fails. Candidates should remain calm in the event of a technical glitch; the computer will automatically stop the time and save the candidates work up to that point. Candidates testing time will not be penalized during the interruption. As stated in the Examinee Agreement and Sign-In (EASI) form, the test proctor is allowed up to 30 minutes to resolve any technical issue. If the resolution of a technical issue takes more than 30 minutes, candidates have the option of rescheduling their exam. In case there are no available seats remaining at the test center during that test administration, ACT has the option of allowing the candidate to test during the next test administration. It is not possible for the FSOT to be administered outside of the three official testing windows. Candidates should be careful to avoid making keystroke errors, which are most frequently made when candidates are editing the essay portion of the FSOT. Keystroke errors cannot be recovered. Candidates will not be allowed to retest based on a keystroke error.

Step 2. Taking the FSOT


Each of the four sections of the FSOT Job Knowledge, Biographic Questionnaire, English Expression, and Written Essay tests is timed. Each test must be completed within the designated time limit, and candidates are not allowed to return to a previous test section if they finish a later section with time remaining. Therefore, it is important that candidates have a strategy for pacing themselves to ensure they will complete each test section in the time allotted.

FSO STORIES FROM THE FIELD


It was Thanksgiving Day in Baghdad. I was accompanying a senior embassy official to a meeting at a TV station outside of the Green Zone on the other side of the city. The convoy needed to escort us to this meeting consisted of a Humvee in front, followed by several armored cars, with a Humvee tank bringing up the rear. The trip across town started with a bullet pinging the rear of the Humvee tank as the convoy left the Green Zone. This set the tone for the day. Our convoy sped along the streets, with the security guys hanging out the front passenger doors with their weapons drawn as we turned corners. One of those corners proved to be too much, and when the armored car in front of ours took it too quickly, one of the security guys fell out . and his vehicle didnt stop to pick him up. Luckily, for our injured colleague, the car behind us stopped for him. Then, at the busiest intersection in the city of Baghdad, our security detail suddenly jumped out of the cars, guns drawn. Apparently, one of our armored cars had stalled and we had to sit perfect targets in the middle of this intersection until our equivalent of a tow truck arrived. At this point, a member of the security detail suggested that we might want to go back to base, but my principal officer shook her head. A half-hour later, the tow truck arrived, and we continued to the TV station minus one armored car for our meeting. But even then, as we were getting out of the car, gunfire sprayed the ground around us. Yes, at that moment, I could have given Jackie Joyner-Kersee a run for her money!
Female Public Diplomacy Officer, 7 years experience

For the multiple-choice test item sections, candidates can easily figure out how much time they should spend on each question to answer the questions in the allotted time. However, this is an individual decision. Some people may not wish to pay attention to the time at all, but may simply prefer to work as quickly and steadily as possible. In the timed essay exam, candidates may wish to spend the early part of the exam organizing their ideas on the topic. Of course, there are many methods for writing an essay. The important thing to realize is that candidates should have a plan for how to organize their time. The tests are designed so that most candidates can finish them in the designated time period. For each section of the exam, candidates should: Read the directions carefully. The urge to quickly scan the directions to the exam and start answering the questions is a natural one, but it is also unwise. It is important to read each question and its answer choices carefully before responding. Skipping over key words or concepts could lead to making mistakes and inadvertently answering an item incorrectly. At times, candidates may be unsure of the answer to a particular question. One strategy is to skip these questions, go on to the questions they can answer, and return to the more difficult questions later while time still remains before exiting that section of the test. NOTE: Once candidates exit a test section, they will not be allowed to return to it. Candidates need to balance their time in answering the items as accurately as possible throughout the test. Because the Job Knowledge and English Expression sections of the FSOT contain multiple-choice questions, it is most advantageous for candidates to answer all of the questions even if they have to guess. There is no penalty for guessing. The number of questions that candidates answer correctly on these two multiple-choice item tests determines their score. If candidates can eliminate one of the answer choices, they have a better chance of choosing the correct answer. One strategy for guessing that should be avoided is choosing answers in a pattern. Standardized tests are designed so that the correct answers to items are randomly distributed and do not follow any guessing patterns. Thus, candidates should avoid using this guessing tactic, because it is likely to be ineffective and detrimental to the candidates scores.

FSO STORIES FROM THE FIELD


My wife and I were in an Eastern European country not long after the fall of the Iron Curtain, when the political, economic, and security situations were all in flux. Our car was stolen. Our visiting friends from the United States were arrested and jailed for no reason. We were harassed by police. Our apartment was burglarized. In the case of the burglary, we had returned from a screening of Its a Wonderful Life, which Id organized at the American Center, to find our apartment ransacked. Among other things, we lost jewelry, family heirlooms, and a large amount of cash Id set aside for our Christmas vacation to Brussels (we were leaving the next morning and ATMs were not to be found). I remember thinking, theres no Uncle Billy coming around with a basket full of cash singing Auld Lang Syne this time. And yet, we left the post two years later with only the fondest of memories, wonderful friends, and experiences including those above that strengthened and enriched our lives in ways from which we continue to benefit. We were able to separate the unfortunate things that happened to us from the rewarding work, rich culture, and welcoming community that we encountered every day of our tour.
Male Public Diplomacy Officer, 20 years experience

When contemplating a strategy for completing the test in the required time, candidates should try to factor in time to review their answers. Candidates may need to return to questions they skipped or reconsider questions about which they were unsure of the answer when they first read them. Keep in mind that all test-taking tips are general. Candidates should determine a test-taking strategy that works best for them.

Knowledge and Skill Areas on FSOT


Proper preparation for taking the FSOT involves much more than studying for a test. It requires that the candidate read widely from many different sources and/or study course work in a number of different fields before taking the test. In the development of the FSOT, a job analysis was conducted of the positions held by Foreign Service Officers to identify the knowledge and skills critical to success on the job. Then, a detailed test blueprint was created. The test blueprint provides an outline of the required knowledge and skill areas and their relative importance to the job. The knowledge and skill areas covered on the FSOT are listed below. Correct grammar, organization, writing strategy, sentence structure, and punctuation required for writing or editing reports. This knowledge area encompasses English expression and language usage skills required for preparing or editing written reports, including correct grammar and good writing at the sentence and paragraph level. United States Government. This knowledge area encompasses a general understanding of the composition and functioning of the federal government, the Constitution and its history, the structure of Congress and its role in foreign affairs, as well as the United States political system and its role in governmental structure, formulation of government policies, and foreign affairs.

FSO STORIES FROM THE FIELD


Our transfer to Jamaica was sudden and thus fairly traumatic for my 11-year-old son. He was leaving a great school, friends, and an embracing embassy community on two weeks notice. I enrolled him in the international school where everybody went. It was a miserable experience as he was surrounded by entitled brats and careless teachers. After months of dinner-table hand-holding, I decided to enroll him at the only local school run by Jesuits. The school reserved two spots each year for non-Jamaican children, and he was accepted to one of them. I then became concerned how his seventh-grade classmates would see him. Would he be an outsider, ostracized, mocked, or marked by his blonde hair, blue eyes, and freckles? To our surprise, the class immediately took him in, turning him into something of a mascot, a welcome status sign for the 12-year-old and 13-year-old students. At the end of the school year, there was a party at one of the students homes a large villa with a swimming pool and acres of lawn. It wasnt until we got there that I realized it wasnt an end-ofthe-school-year party. It was a farewell bash thrown for my blue-eyed, freckled, blonde-haired son.
Female Management Officer, 31 years experience

United States History, Society, Customs, and Culture. This knowledge area encompasses an understanding of major events, institutions, and movements in national history, including political and economic history, as well as national customs and culture, social issues and trends, and the influence of U.S. society and culture on foreign policy and foreign affairs. World History and Geography. This knowledge area encompasses a general understanding of significant world historical events, issues, and developments, including their impact on U.S. foreign policy, as well as knowledge of world geography and its relationship to U.S. foreign policy. Economics. This knowledge area encompasses an understanding of basic economic principles, as well as a general understanding of economic issues and the economic system of the United States. Mathematics and Statistics. This knowledge area encompasses a general understanding of basic mathematical and statistical procedures. Items requiring calculations may be included. Management Principles, Psychology, and Human Behavior. This knowledge area encompasses a general understanding of basic management and supervisory techniques and methods. It includes knowledge of human psychology and behavior, leadership, motivational strategies, and equal employment practices. Communications. This knowledge area encompasses a general understanding of the principles of effective communication and publicspeaking techniques, as well as general knowledge of public media, media relations, and the goals and techniques of public diplomacy and their use to support work functions. Computers and the Internet. This knowledge area encompasses a general understanding of basic computer operations such as word processing, databases, spreadsheets, and using e-mail and the Internet.

10

FSO STORIES FROM THE FIELD


Sometimes we forget that our children need to learn American culture while theyre absorbing the foreign ones. When my youngest son was six years old, the first mall (quite small, with one anchor and some fast-food places, but the first assortment of commercial enterprises in one building under one roof) opened in Costa Rica. The older children decided an outing for pizza was just the ticket, so we went. Our six-year-old son finished first and, of course, got fidgety. I asked his older brother to take him for a walk in the mall to which our six-year-old son responded, Whats a mall? Thats when I knew a tour in Washington, DC, was in order.
Female Management Officer, 31 years experience

Suggested Courses of Study


Success on the FSOT is not necessarily dependent on a specific course of study. However, the curriculum of the following college-level courses should prepare a candidate for the content of the test. The names of the courses are general and may differ from institution to institution. English Composition/Rhetoric American History American Studies (including cultural and social history) American Political Thought United States Political System American Economic History Introduction to Economics (micro and macro) World History (Western and non-Western) World Geography International Economics World Religions Introduction to Statistics Introduction to Management Principles Intercultural Communication Mass Communication Psychology

How to Get Started


A booklet entitled Guide to the Foreign Service Officer Selection Process can be downloaded (in PDF format) from the registration website at careers.state.gov. To request a hard copy of the guide, please e-mail state@socent.org. Online registration for the FSOT is available during the registration period at the following website address: careers.state.gov.

11

FSO STORIES FROM THE FIELD


In 2003, a diplomatic convoy that I was traveling in was hit by a roadside bomb. I heard a loud explosion and looked out the vehicles rear window in time to see the SUV behind mine fall to the ground and break apart. Within seconds, scores of people came running to the scene. Thanks to the quick thinking of my driver, who quickly turned the vehicle around and sped to a safe haven, I survived. Unfortunately, several people were killed. The incident made me keenly aware of the dangers that Foreign Service Officers face every day as we advance U.S. interests abroad. It also strengthened my resolve to promote U.S. values and address misconceptions about the United States.
Male Public Diplomacy Officer, 18 years experience

Once the Test Session Is Over


Candidates are free to leave the test center once they have finished taking the FSOT. Because individual candidates may have begun their exams at different times, they also may be leaving as others are still testing. Please be considerate and try to minimize any disruptions. Bring any testing irregularities to the proctors attention immediately or, at the very least, before leaving the test center. ACT and the Department of State work hard to provide candidates with appropriate testing environments and the proper tools, and they need to know if anything fails to meet appropriate standards. Filing a complaint with the proctor will NOT have any bearing on whether a candidate passes the FSOT. Candidates should mark their calendars for 3-4 weeks after the test to remind them that their test results should be received by that time. If candidates dont receive an e-mail with their test results from ACT within that 3- to 4-week time frame, then they are responsible for following up. To prevent difficulties in receiving results, candidates should: Check their spam folders. Make sure their spam filters accept e-mails from FSOT@act.org. Make sure the e-mail address they provided to ACT up to a year previously is the same e-mail they are currently using. If not, they should contact ACT to update their e-mail addresses. Print any e-mails received from ACT after the FSOT has been taken, since candidates may not be able to access them later.

Neither ACT nor the Department of State will be responsible for the failure of candidates Internet servers to deliver e-mail in a timely fashion.

12

FSO STORIES FROM THE FIELD


We were overseas with our five-month-old baby our firstborn when my husbands elderly father suffered a paralyzing stroke. There were eight weeks left until I completed the minimum amount of time in-country required to constitute a full tour, so my husband returned to the United States with our baby while I stayed at Post. It wasnt until I was waving goodbye to them at the airport that I realized that those eight weeks would constitute over one third of my childs life thus far and that she wouldnt remember who I was when she saw me next. Female Economics Officer, 23 years experience

Step 3. Qualifications Evaluation Panel Review Process


The Qualifications Evaluation Panel (QEP) review complements the results achieved on the FSOT and the Oral Assessment by looking at a candidates total file: the FSOT and Written Essay scores, work history, education, and personal experiences, as well as any language ability validated by the Foreign Service Institute. Beyond relying on the test scores alone, FSOs screen candidates for additional jobrelated qualities that indicate a candidates ability to perform Foreign Service work successfully. Candidates who pass the FSOT are invited to submit Personal Narratives (PNs) in which they answer questions describing the knowledge, skills, and abilities they would bring to the Foreign Service. Candidates will be asked to address the following six areas: Leadership, Interpersonal Skills, Communication Skills, Management Skills, Intellectual Skills, and Substantive Knowledge. Candidates are allowed 3 weeks after receiving their FSOT results to submit their PNs; there is no flexibility in this deadline. The invitation to complete the Personal Narrative (PN) survey is included in the Passing letter received from ACT. To gain access to the Passing letter, candidates must log-in to their account, as instructed in the e-mail from ACT, and download a PDF of their FSOT results letter. A link is provided in the Passing letter for candidates to launch the survey to complete their PNs. Candidates should check their spam filters and contact ACT if they have not received the e-mail notification that they passed the FSOT with the invitation to submit their PNs. NOTE: If candidates fail to submit their PNs, their candidacy will be terminated. Candidates may wish to keep the following considerations in mind when drafting their PNs: Respond appropriately to the questions asked. A candidate may have a great story that highlights his or her unique qualities, but if the response does not directly address the question, the candidate will receive a lower score. The PNs allow candidates to draw attention not only to what they have done, but also how they did it, why it mattered, and what effect it had. Simple lists of accomplishments, recaps of candidates resumes, or theories learned in school are not helpful.

FSO STORIES FROM THE FIELD


My kids grew up thinking Mylar on the windows, safe rooms in houses, mirror sweeps of the undercarriages of vehicles, testing for explosive residue on car engines, and weekly emergency radio check-ins were normal and this was before 9/11. We spent four years at one post that had a school which, although it had been fully accredited by the New England Association of Schools and Colleges, proved to be mediocre. When we returned to the U.S. after that tour, my daughter made the unhappy discovery that she was a year behind her classmates in math. That said, my children could have successfully negotiated their way through customs by themselves anywhere in the world by age seven.

Female Economic Officer, 23 years experience

13

The QEP panels are composed of experienced Foreign Service Officers who have read the PNs from hundreds of candidates and are familiar with the type of experience a candidate acquires when spending two weeks in Africa for a graduate school project or working as a summer intern in the State Department. While candidates should not be shy about reporting their actual accomplishments, inflating the facts or their work positions is counterproductive and very likely to work against them. Candidates should not make assumptions about the types of responses the Department of State is looking for or the type of candidate it wishes to hire. While most candidates have had some overseas experience e.g., travel, study, or business many have not. It is possible that a candidates summer job at a local animal shelter could provide a better response to a PN question than the experience acquired during study abroad. Before submitting their PNs, candidates should make sure they have provided responses that not only answer the specific question, but also are appropriate to the career track selected when they registered. Candidates are rank-ordered with other candidates in the same career track. If the consular career track is selected, it is not likely that a candidate will be ranked highly if he or she only provided responses demonstrating the potential to be an economic officer. During every test administration, the QEP panels discover several PN submissions many of which are very high quality that indicate the candidate is applying for a Foreign Service Officer position as, for example, a political officer, when the actual career track selected was consular. This mistake is particularly common for candidates who attained the QEP stage on another occasion and are simply recycling their previous responses. Neither the Department of State nor ACT has the ability to change candidates career tracks once they have been selected during the registration process.

The test administrator will forward candidates files to the Department of State for the QEP review, where each file will be evaluated by panels of experienced Foreign Service Officers and assigned a numerical score. It is important to note that there is no Pass or Fail in the QEP review. Rather, candidates receive a relative ranking compared to other candidates in the same career track. Thus, the higher a candidates score, the more likely the candidate will be invited to take the Oral Assessment. There is no preset cut-off score.

Step 4: Invitation to Take the Oral Assessment


The Department of State will issue invitations to take the Oral Assessment based on its anticipated hiring needs. Candidates should understand that the percentage of candidates who passed the FSOT and those who are invited to take the Oral Assessment varies dramatically over time. Many candidates with excellent qualifications may receive an invitation to take the Oral Assessment at a time when a greater number of new positions are available at the Department of State. On the other hand, candidates with excellent qualifications may not be invited to take the Oral Assessment when the Department of States hiring targets are lower. The process is very competitive, and many candidates repeat the selection process. Additional information about the Oral Assessment is provided in the last section of this Guide (please see pages 63-64).

14

JOB KNOWLEDGE TEST SECTION


The Job Knowledge section of the FSOT contains questions that cover several major content areas, such as the U.S. Government and Political System, U.S. and World History, Geography, Economics, and Math and Statistics. It consists of 60 questions administered in 40 minutes. The knowledge and skill areas covered in the Job Knowledge section of the test were described previously in this Study Guide. (NOTE: English language skills are tested in the separate English Expression and Written Essay sections of the FSOT, not in the Job Knowledge section of the test.) The Job Knowledge test consists of questions that represent the knowledge and skill areas that are equally important across all five career tracks. There are 60 sample items provided below. After circling the answer to each of the 60 sample questions given below, candidates can check their answers and review the rationales for the answer to each question in the next section of this Guide.

Sample Questions for the Job Knowledge Test


DIRECTIONS: Each question or incomplete statement below is followed by four suggested answers or completions. Read each item and select the one answer that is best in each case.
1. In 1964, the U.S. Congress passed the Gulf of Tonkin resolution authorizing: A. the partition of Vietnam. B. an American invasion of North Vietnam. C. an American military alliance with South Vietnam. D. presidential discretion in defending U.S. forces in and around Vietnam. 5. 2. The battle most often credited with turning the tide in the Pacific Theatre during World War II was the Battle of: A. B. C. D. 3. Leyte Gulf. Iwo Jima. Midway. Okinawa. 4. The phrase priming the pump through increased government spending is most closely associated with which of the following economic theories? A. B. C. D. Keynesianism Monetarism Supply-side economics Transition economics

An economys gross domestic product (GDP) is $30,000,000 in year 1 and $31,500,000 in year 2. If the population were 100,000 in year 1 and 105,000 in year 2, the growth rate of GDP per capita in this economy would be: A. B. C. D. 0%. 2%. 3%. 5%.

Irans location allows it to exercise control over which strategic international maritime pathway? A. B. C. D. Suez Canal Strait of Hormuz Gulf of Bahrain Gulf of Aden

6.

A wide span of management control is most likely to result in: A. B. C. D. overly tight supervision of employees. isolation of top managers. increased labor costs. lack of control of employees.

15

7.

Judging by their domain names, which of the following fictitious Web pages would be LEAST likely to provide objective information about the stock market? A. B. C. D. Stocks.org WallStreet.gov WallStreet.net Stocks.com

14.

Which type of tax is an example of a progressive tax system in the United States? A. B. C. D. Sales tax Income tax Capital gains tax Social Security (FICA) payroll tax

15.

The response most frequently given to a survey question is called the: A. B. C. D. median. mode. variance. standard deviation.

8.

Which committee in the U.S. House of Representatives is responsible for making funding decisions regarding expenditures from the Federal Treasury? A. B. C. D. Appropriations Committee Ways and Means Committee Committee on Standards of Official Conduct Rules Committee 16.

A typical initial group reaction to conflict is: A. B. C. D. open communication. trust building. avoidance. problem solving.

9.

In recent years, the term dtente has typically referred to: A. Iraqs military incursion into Kuwait. B. the U.S. policy of containment during the Cold War. C. the Paris Peace Treaty signed to end the war in Vietnam. D. peaceful coexistence between the United States and the USSR. 17.

The Equal Pay Act of 1963 prohibits which type of discrimination? A. B. C. D. Sex Race Age Disability

10.

During the 1919 Paris Peace Conference, which country initiated the creation of the League of Nations? A. B. C. D. Germany France Great Britain The United States

18.

Which term refers to a pithy phrase or quotation that captures an essential point of a particular speech? A. B. C. D. Sound bite Paraphrase Maxim Motto

11.

Which nation was formerly known as East Pakistan? A. B. C. D. Afghanistan Bangladesh Myanmar Sri Lanka

19.

Which term relates to a Web site or online document that serves as a repository of information about a topic and allows knowledgeable contributors to edit or add to its content? A. B. C. D. Webcast Social network Wiki Vlog

12.

Stephen Cranes powerful novel about the U.S. Civil War is titled: A. B. C. D. The Thin Red Line. To Hell and Back. The Red Badge of Courage. Battle Cry. 20.

13.

Which country was the first to be freed from Communist authority after the fall of the Berlin Wall in 1989? A. B. C. D. Hungary Yugoslavia East Germany Ukraine 16

In 1950, President Harry Truman justified sending U.S. troops into Korea without seeking formal congressional approval based on which organizations passage of a resolution and involvement in the conflict? A. B. C. D. North Atlantic Treaty Organization (NATO) United Nations Security Council League of Nations European Union

21.

The United States moved closer to civil war when bloody unrest broke out between 19th century abolitionists and slavery supporters in what territory? A. B. C. D. California Kansas Oklahoma Minnesota

26.

The best way for a manager to motivate employees is to: A. set an example of the expected behaviors through personal actions. B. make a disciplinary example of one or more employees who are not meeting expected behavior. C. offer recognition, such as employee-of-themonth designations, for expected behavior. D. offer increased pay or bonuses for expected behavior.

22.

Jesse Owens four gold medals in the 1936 Olympics were significant because: A. they were the most ever won by an athlete in one Olympics. B. they were the first Olympic medals won by an athlete of color. C. Owens was an outspoken Civil Rights leader. D. Owens success challenged German racial theories. 27.

Which phrase best describes the term cloud computing? A. Technology that processes data using a light source B. A system that provides infinite data storage C. A complete virtual world, such as Second Life D. Services that allow users to store data and access applications online

23.

The Hungarian uprising of October 1956 might have received more international support and contributed to an earlier demise of the Soviet Union if not for the simultaneous occurrence of: A. Khrushchevs Secret Speech to the 20th Party Congress. B. the construction of the Berlin Wall. C. the Suez Canal crisis. D. the Boer War.

28.

The U.S. Senate procedure that permits the termination of a filibuster by a supermajority vote of 60 senators is known as: A. B. C. D. recommital. enrollment. unanimous consent. cloture.

24.

One outgrowth of Title IX legislation was a significant increase in: A. the participation of women in school sports. B. African Americans in elected office. C. female commissioned officers in the armed services. D. disabled workers in government workplaces.

29.

Which amendment to the U.S. Constitution guarantees the right to keep and bear arms? A. B. C. D. First Second Third Fourth

25.

The employees of a company have a current average salary of $30,000 per year, and the standard deviation is $1,000. In response to a recent strike, salaries are to be increased by $3,000 per year for each employee. After the raise, what will be the new average salary and the new standard deviation? A. B. C. D. Average Salary $33,000 $33,000 $30,000 $30,000 Standard Deviation $1,300 $1,000 $4,000 $1,100

30.

Which three countries exercise de facto administrative and military control over Kashmir? A. B. C. D. India, China, and Pakistan India, Pakistan, and Afghanistan Pakistan, Afghanistan, and China India, Nepal, and Pakistan

31.

Supply-side economics is a concept that advocates for: A. B. C. D. higher tariffs. stock splitting. government subsidies. lower marginal tax rates.

17

32.

The growth of global news media has prompted contentions that media organizations now play a significant role in forming U.S. foreign policy. This concept is called: A. B. C. D. media globalization. the network society. the CNN effect. global news flow.

38.

In 2002, the United States established its first permanent military base in which African country? A. B. C. D. Nigeria Djibouti South Africa Kenya

39.

33.

Which term refers to a malicious computer program that can replicate itself without attaching itself to an existing program, typically using computer networks and the Internet to spread to vulnerable computers and cause harm to the network? A. B. C. D. Virus Rootkit Worm Trojan horse

A wholesaler offers its best customers a 55% discount on 4/5 of its merchandise and a 70% discount on the remaining 1/5 of its merchandise. What is the average discount offered to the best customers? A. B. C. D. 58% 62.5% 67% 79%

40.

34.

Which of the following committees, including members from both the House and the Senate, are used to reconcile differences between bills that have been passed by both chambers? A. B. C. D. Standing committees Conference committees Authorizing committees Select committees

An embassy employee is to receive a monetary bonus for performing exemplary work on a particular project. Which of the following scenarios is the best way to inform the employee of this recognition? A. An announcement is made at a team meeting that the employee and a few others are receiving a monetary reward for their good work. B. The immediate supervisor personally informs the employee of the monetary bonus and acknowledges the employees accomplishment in a timely manner. C. A memo from the Ambassador informs the employee of the monetary bonus and expresses appreciation for the good performance. D. The employee receives notice of the bonus during a formal annual performance review.

35.

Which U.S. government agency publishes a weekly updated World Factbook on the countries of the globe, including their populations, major cities, geography, economy, government, public health, and culture? A. B. C. D. Agency for International Development Department of State, Bureau of Public Affairs Overseas Private Investment Corporation Central Intelligence Agency 41.

Which public speaking technique would hold the audiences attention most effectively? A. Distributing printed copies of the speech in advance of the presentation B. Occasionally using pauses and silence at specific points during the speech C. Using highly technical language to describe different activities and organizations D. Using slang language to appear more in touch with current trends

36.

The 1997 Kyoto Protocol is primarily designed to address the problem of: A. B. C. D. climate change. ocean pollution. the trade in endangered species. child labor.

37.

U.S. President Theodore Roosevelt won the Nobel Peace Prize for taking a lead role in negotiating a treaty between which two warring nations that would become future world powers? A. B. C. D. Russia and Japan France and Germany England and Germany Austria and Italy

18

42.

The U.S. Senate failed to approve a resolution to ratify the Treaty of Versailles in large part because politicians: A. separated into factions and could not achieve a two-thirds majority. B. expected the League of Nations to make the treaty irrelevant. C. demanded the partition of the AustroHungarian Empire. D. opposed President Wilsons plan to amend the treaty.

47.

Three employees earn $8 per hour, $9 per hour, and $10 per hour, respectively. If the wages of all three employees are increased by $1 per hour, the range will increase by: A. B. C. D. 0. 1. 2. 3.

48.

Employees intrinsic motivation to perform good work is most likely to increase when a job is enriched by managements effort to: A. provide detailed instructions to all employees on how to perform their job duties. B. delegate project tasks to employees primarily on their availability to perform the work. C. increase base salaries and benefits for employees based on their length of service. D. assist employees to learn new skills based on their interests and to promote job growth.

43.

Optical character recognition (OCR) is a technology that performs what valuable function? A. It identifies typographical errors in data files and automatically corrects them. B. It allows users to scan remote screens for data and enters the data into a database. C. It enables a spell-check feature to verify specialized vocabulary in text files. D. It scans printed pages and translates them into computerized text files.

49.

Libertarianism is a political philosophy that is based on which of the following principles? A. Access to libraries is essential to a working democracy. B. Government should provide considerable constraints on the behavior of its citizens. C. An individuals obligation to the community must take precedence over personal desires. D. Property rights are essential and primary.

44.

Senators often attach riders to bills being considered in the U.S. Senate. Which of the following phrases best describes a rider? A. A substitute measure replacing the original bill B. A provision that exempts the bill from most Senate rules C. An amendment that is not germane to the bill D. A provision that returns the bill to committee 50.

45.

The most effective approach to project management is to structure the project team as: A. a group of independent contributors each with a clear set of tasks to complete. B. a group of outside experts each focused on contributing to the project based on their expertise. C. experienced, interdependent group members with clearly defined individual project responsibilities. D. experienced, interdependent group members who share collective responsibility for all project tasks.

Congress is bound by Article I, Section 8 of the U.S. Constitution to exercise only those powers that are stated in the Constitution. What are these powers called? A. B. C. D. Enumerated powers Manifest powers Executive powers Congressional powers

51.

Which e-mail address field is used to conceal the recipients identity so that only the recipient with the address listed in that field knows that he or she received the e-mail? A. B. C. D. To From Cc Bcc

46.

Which idea or practice was challenged by the growing immigrant populations in the United States after 1960? A. B. C. D. Bilingual education Affirmative action Multiculturalism Assimilationism 19

52.

Transformational leadership is most likely to be effective in organizations that: A. need to adapt to a changing environment. B. are small and have few departments and divisions. C. are based on a bureaucratic structure. D. are vertically structured.

57.

U.S. foreign policy toward Africa was altered when the African Growth and Opportunity Act (AGOA) was passed by Congress and signed into law by President Clinton in May 2000. This new policy represented a shift from emphasizing: A. U.S. aid to emphasizing U.S. trade. B. U.S. trade to emphasizing U.S. aid. C. U.S. aid to emphasizing more concern for protecting African human rights. D. U.S. trade to emphasizing more concern for protecting African human rights.

53.

The author of Walden and Civil Disobedience was: A. B. C. D. John Locke. Jean-Jacques Rousseau. Henry David Thoreau. Thomas Hobbes.

58.

Managers who have an ethnocentric mindset believe that: A. it is valuable to use host-country managers in certain key positions. B. home-country managers have superior management skills and knowledge. C. compensation for host- and home-country managers should be the same. D. host-country managers have the best approach to local management.

54.

What is a fundamental purpose of performance feedback provided by a supervisor to an employee? A. To reprimand an employee for unsatisfactory performance B. To ensure employees are aware of their weaknesses and to take corrective action C. To offer a professional judgment on an employees capabilities D. To support professional development and achievement

59.

Which of the following rights or powers represents a check and balance on judicial power? A. The Presidents power to remove judges B. The Presidents right to expand the number of federal courts, with the approval of the Senate C. The Senates power to confirm the Presidents judicial appointments D. Congress power to approve the Presidents judicial appointments

55.

Which term refers to a program that secretly, and often maliciously, saves or transmits information that a user types into a file? A. B. C. D. Spider Keylogger Phisher Broadcaster 60.

56.

A U.S. businessperson seeks advice from a U.S. Foreign Service Officer (FSO) in a country notorious for governmental corruption. The businessperson has been told that to win a lucrative government contract a commission must be paid to officials involved in granting the contract. What advice should the FSO give to the businessperson? A. It is permissible to pay the commission if this is a common and accepted practice in that country. B. It is permissible to pay the commission if the laws of the host country prohibiting such commissions are not enforced. C. Paying such a commission is a violation of U.S. law. D. No advice should be given by U.S. officials regarding a private business deal.

A Foreign Service Officer (FSO) has been assigned to work in a country with widely varying terrain. The FSO must orient a group to the nearby geographical features, including the shore to the south, mountains to the north, and rough brush to the east. What would be the most logical organizational pattern to use in a presentation to orient the group to the region? A. B. C. D. Spatial Categorical Problem solution Causal

20

Rationales for Sample Job Knowledge Questions


1. Knowledge tested: U.S. Government: Congress and its role in foreign affairs A. Vietnam was effectively partitioned in 1954 as part of the Geneva Convention ending the First Indochina War. The Gulf of Tonkin resolution makes no mention of partitioning. B. The Gulf of Tonkin resolution authorizes military force only for defensive purposes. C. The Gulf of Tonkin resolution is brief and does not discuss any long-term military alliance between the United States and South Vietnam. D. Correct answer. The Gulf of Tonkin resolution received congressional approval, authorizing President Johnson to use military force in Southeast Asia for defensive purposes. The text of the resolution states That the Congress approves and supports the determination of the President, as Commander in Chief, to take all necessary measures to repel any armed attack against the forces of the United States and to prevent further aggression. 2. Knowledge tested: U.S. History: Major historical events (e.g., slavery, Declaration of Independence, Civil War, Great Depression, etc.) A. Fought in 1944, the Battle of Leyte Gulf dealt a major defeat to the Japanese navy, but it came long after the Japanese forces had been forced to retreat closer and closer to their home islands. Following this battle, the Japanese navy vessels never left their home bases again during World War II. B. Although Iwo Jima was a hard fought battle with high casualty rates, the tactical advantage gained by capturing the island was relatively small. It took place in 1945, 6 months before the dropping of the atomic bomb, and was too late in the war to be considered a moment that turned the tide. C. Correct answer. The Battle of Midway, fought in 1942 near the U.S. mid-Pacific base at Midway Atoll, was considered to be an important turning point in the Pacific Theatre during World War II. Six months after Japans attack on Pearl Harbor, the American naval fleet inflicted a major blow to the Japanese carrier fleet, limiting Japans ability thereafter to mount a major offensive anywhere else in the Pacific. D. Another hard fought battle with high casualty rates, the battle for Okinawa took place in 1945, 3 months before the dropping of the atomic bomb, but it was too late in the war to be considered a moment that turned the tide. 3. Knowledge tested: World Geography: Location of countries and their physical features A. The Suez Canal is an artificial waterway in Egypt that connects the Mediterranean Sea with the Red Sea. It is under Egyptian control. B. Correct answer. The Strait of Hormuz is located between Oman and Iran and connects the Persian Gulf with the Gulf of Oman and the Arabian Sea. In recent years, Iran has threatened to close the Strait of Hormuz, which would affect nearly 20% of oil traded worldwide. C. The Gulf of Bahrain is an inlet of the Persian Gulf on the east coast of Saudi Arabia. It surrounds the islands of Bahrain. D. Unofficially known as Pirate Alley due to acts of piracy, the Gulf of Aden is located in the Arabian Sea and is the southern entry point to the Suez Canal. 4. Knowledge tested: U.S. economic principles, such as supply and demand A. Correct answer. Keynesian economics advocates government and public sector spending to boost the economy. It is believed that higher government spending in a recession can help the economy recover more quickly. B. Monetarism emphasizes the importance of controlling the money supply to control inflation. Price stability, wherein government controls the circulation of currency, is at the center of monetarism. C. Supply-side economics holds that lower tax rates and less government regulation will help boost the economy. D. Transitional economics focuses on moving from a centrally planned economy to a free market, wherein the market, rather than the government, sets prices. 5. Knowledge tested: Mathematics: Fundamental principles of mathematics (e.g., multiplication, division; calculation of growth rates, etc.) A. Correct answer. GDP per capita in year 1 = $30,000,000 divided by 100,000 = $300. GDP per capita in year 2 = $31,500,000 divided by 105,000 = $300. (300 300)/300 = 0%. Thus, there was no growth in GDP per capita. B. See A. C. See A. D. See A (5% is the growth in GDP from year 1 to year 2, not the growth in GDP per capita.)

21

6.

Knowledge tested: Management Principles, Psychology, and Human Behavior: Knowledge of basic leadership principles, such as modeling A. Overly tight supervision of employees is more likely to be a problem when managers have a narrow span of control. B. A wide span of management control is more likely to lead to employees feeling ignored or isolated (rather than top managers feeling isolated), because the manager has more employees to supervise and limited time to give individual attention to employees. C. Larger spans of control allow an organization to operate with fewer layers of middle management, which can reduce labor costs. D. Correct Answer. Span of control relates to the number of employees who report to a manager. Narrow spans of control imply close supervision. Wider spans of control require more autonomy on the part of employees. The wider the span of control, the more likely a supervisor is to lose control over employees.

8.

Knowledge tested: U.S. Government: Understanding of the composition of the federal government (e.g., branches, appropriations process, Supreme Court, etc.) A. Correct answer. In the U.S. House of Representatives, the Appropriations Committee appropriates funding from the Federal Treasury to pay for programs authorized by the chamber's various authorizing committees. B. The Ways and Means Committee is responsible for making decisions related to taxes and taxation. C. The Committee on Standards of Official Conduct is responsible for policing the behavior of members, considering charges related to ethics violations. D. The House Rules Committee is responsible for controlling access to the floor for consideration of most bills; it makes decisions regarding the amount of time allotted for debate and determines whether a bill will be subject to amendments.

7.

Knowledge tested: Communications: Sources of information (e.g., newspapers, television, radio, published documents, etc.) A. The .org top-level Internet domain was originally intended to identify nonprofit organizations. As such, these organizations may promote a particular point of view, but they do not typically stand to gain materially from the information they provide. B. The .gov top-level Internet domain identifies an office or department of the U.S. government, which has the mission of providing public service and reliable information to citizens. C. The .net Internet top-level domain was originally intended for use by computer network companies. It is considered to be less clearly commercial than the .com domain, but is reportedly less objective than the .org domain. D. Correct answer. The .com Internet top-level domain usually identifies a commercial Web site that was originally intended to be used by for-profit business entities. As such, the entities posted on this site may have a motiveto sell stocks or investment advicewhich is likely to indicate that information provided on the site has a subjective/partisan point of view. Thus, the .com domain is deemed least likely to provide objective information.

9.

Knowledge tested: U.S. Government: National interests and objectives pursued abroad (e.g., human rights, terrorism, crime, U.S. trade, etc.) A. From the French word for relaxation, dtente is not used to refer to invasions such as Iraq's incursion into Kuwait. B. Containment did not involve relaxation, but rather a vigilance to head off Communist efforts to expand their scope of influence. C. Dtente is typically used in a general sense to indicate a broad but informal easing of tensions, not a formal treaty to end hostilities. D. Correct answer. Beginning in the early 1970s, both the United States and the Soviet Union undertook a number of measures to ease the tensions of the Cold War and facilitate peaceful coexistence, including the Strategic Arms Limitation Talks (SALT I), Helsinki Accords, and the Anti-Ballistics Missile (ABM) Treaty.

22

10.

Knowledge tested: U.S. History: Influence of U.S. political system and society on foreign affairs A. Germany became a member of the League of Nations but did not propose it originally. B. France supported the creation of a League of Nations but did not propose it originally. C. Great Britain supported the creation of a League of Nations but did not propose it originally. D. Correct answer. U.S. President Woodrow Wilson proposed the creation of the League of Nations before the end of World War I as part of his Fourteen Points for Peace. Wilsons proposal to form the League was approved at the Paris Peace Conference, and the League became Part I of the Treaty of Versailles, with the goal of building a lasting peace after the war. The League was replaced by the United Nations after World War II.

13.

Knowledge tested: World History and Geography: Key historical events A. The Soviet Union reduced its involvement in Hungary by signing an agreement in April 1989 to withdraw its forces by June 1991. B. The Socialist Federal Republic of Yugoslavia comprised of Bosnia, Herzegovina, Croatia, Macedonia, Montenegro, Serbia, and Slovenia dissolved in 1992. C. Correct answer. The Berlin Wall was considered to have fallen on November 9, 1989, when Communist authorities of the German Democratic Republic (GDR) announced the removal of travel restrictions, allowing East Germans to cross into democratic West Berlin. D. The Ukrainian parliament declared Ukraine an independent democratic state in August 1991.

11.

Knowledge tested: World History and Geography A. Afghanistan was founded in 1747. The country served as a buffer between the British and Russian Empires until it won independence from British control in 1919. B. Correct answer. East Pakistan became the nation of Bangladesh in 1971. The new country formed a parliamentary democracy under a 1972 constitution. C. Myanmar is also known as Burma. After 1989, the military authorities in Burma promoted the name Myanmar as a conventional name for their state. D. Ceylon is one of the more widely known earlier names for Sri Lanka.

14.

Knowledge tested: U.S. economic issues A. The sales tax is an indirect tax that a seller collects from a buyer and, in turn, pays the government. A sales tax is not related to how much a person earns; it is simply the percentage rate applied to the taxable sale price of a good or service. Since everyone pays the same sales tax for a good or service, regardless of income, it is not a progressive tax. B. Correct answer. In the United States, a progressive tax is one by which the tax rate increases as the taxable base amount grows. It is typically applied in reference to personal income taxes. Progressive taxes attempt to determine the amount of taxes paid based on a persons ability to pay. Persons with lower incomes would be taxed at a lower tax rate, and persons with higher incomes would be taxed at a higher tax rate. C. The capital gains tax applies to the profit obtained by the sale of an asset, such as a stock, bought at a lower price. D. Social Security payroll taxes are collected under the authority of the Federal Insurance Contributions Act (FICA). The FICA payroll tax paid by individuals and employers funds the federal Social Security program. Generally, the FICA Social Security payroll tax is imposed equally on employers and employees.

12.

Knowledge tested: U.S. History, customs, and culture (e.g., traditions, life-styles, education, arts) A. The Thin Red Line is a fictional novel by James Jones about World War II. B. To Hell and Back is a first-hand account by Audie Murphy of life as a soldier during World War II. Murphy was a highly decorated soldier and recipient of the Congressional Medal of Honor. C. Correct answer. The Red Badge of Courage is Stephen Cranes fictional account of a young Union soldier who retreats from battle during the Civil War and covets a red badge of courage as redemption for his lack of courage. In the end, the young private transcends his fear on the battlefield and becomes a courageous soldier. D. Battle Cry was a novel based loosely upon Leon Uris World War II experiences in the Marines.

23

15.

Knowledge tested: Mathematics and Statistics: Basic knowledge of statistics (e.g., mean, mode, standard deviation, etc.) A. The median is the value in the distribution at the 50th percentile of all values. The median of a finite list of numbers also can be found by arranging all of the values from the lowest value to the highest value and picking the middle value. B. Correct answer. The mode is defined as the most frequently occurring value in a distribution. C. The variance is a measure of the spread of the distribution around the mean. D. The standard deviation is the square root of the variance, so it is also a measure of the spread of the distribution around the mean.

18.

Knowledge tested: Communications: Public speaking techniques A. Correct Answer. Bite conveys the shortness of the phrase or quotation, and sound reveals its origins in newscasting. Thus, sound bite denotes a short clip from a speech that captures its essential point. B. A paraphrase is not necessarily short and may not be taken from the original speech. Rather, a paraphrase is a summary or restatement of the meaning of a text or passage using another persons words (not the actual wording from the original source). C. While a maxim is short and pithy, it is not linked to the wording used in a particular speech. Rather, it is a familiar phrase that could be employed in countless speeches. D. A motto is a short phrase that sums up the abiding theme or values of an organization or institution, not the point of a speech on a particular occasion.

16.

Knowledge tested: Management Principles, Psychology, and Human Behavior A. While groups should strive to achieve open communication, it is not the most typical way of handling conflicts when they initially arise in a group setting. B. Conflicts initially can cause people to respond defensively rather than gravitating toward building trust, which typically occurs over time. C. Correct Answer. Because conflict can conjure up strong emotions, a typical initial reaction is to avoid the conflict. Avoidance in a group meeting may be used initially to delay dealing with the conflict until it can be discussed in a more rational way and resolved. D. If handled constructively, groups can work through conflicts and learn to solve problems effectively over time. However, this is not a typical initial group reaction to conflict.

19.

Knowledge tested: Computers and the Internet: Knowledge of basic computer functions, E-mail functions, and the Internet (World-Wide Web) A. A Webcast is a presentation streamed over the Internet for training purposes or to communicate a message to a group of participants. It does not allow different individuals viewing the presentation to edit the streaming message. B. A social network describes a set of social interactions shared with different individuals through social media and Web-based sites. Examples of social networks include Twitter, which is an instant messaging system that allows users to send text messages to followers in their social network, and Facebook, which is a Web-based social networking site that allows users to create and share personal profiles with their friends and colleagues as designated by their privacy settings. C. Correct Answer. Wiki is a Web site that serves as a repository of information about a stated topic and allows knowledgeable contributors to edit or add to its content. D. A Vlog is a blog in a video chronicle format of a persons life, thoughts, opinions, and interests that are likely to be viewed on a site like YouTube.

17.

Knowledge tested: Management Principles: EEO policies, regulations, and laws A. Correct Answer. The Equal Pay Act of 1963 makes it illegal to pay different wages to men and women if they perform equal work in the same workplace. The law also makes it illegal to retaliate against a person because the person complained about discrimination, filed a charge of discrimination, or participated in an employment discrimination investigation or lawsuit. B. Title VII of the Civil Rights Act of 1964 (Title VII) prohibits race discrimination (among other forms of discrimination). C. The Age Discrimination in Employment Act of 1967 (ADEA) prohibits age discrimination. D. Title I of the Americans with Disabilities Act of 1990 (ADA) prohibits disability discrimination.

24

20.

Knowledge tested: U.S. Government: Understanding of relationships and interactions among components of the federal government A. NATO was not involved in the Korean War. B. Correct Answer. Article 53 of the United Nations Charter allows the use of military force in a regional context as a response to aggression. The U.N. Security Council unanimously approved a resolution condemning the invasion by the Democratic Peoples Republic of Korea (or North Korea) into the Republic of Korea (South Korea), and President Truman sent U.S. troops to join the U.N. military effort without seeking a formal declaration of war from Congress. C. The United Nations replaced the League of Nations after World War II. D. The European Union is an economic and political entity.

22.

Knowledge tested: U.S. History: Relationships between U.S. society and other countries A. Finnish national hero, Paavo Nurmi won nine gold medals in three Olympics in the 1920s, before Owens Olympic achievement. Nurmi won five of his gold medals at the 1924 Summer Olympics held in Paris, which is still the highest total of gold medals won by a trackand-field athlete at one Olympics in the history of the Games. B. At the 1908 Olympics, John Taylor was the first African American to receive a medal. C. Owens was not an outspoken Civil Rights leader in 1936 because the Civil Rights movement had not yet begun. D. Correct Answer. James Cleveland "Jesse" Owens was the most successful athlete at the 1936 Olympics in Berlin, Germany. His success was historically notable because it was in direct contradiction to the Aryan racial supremacy theories espoused by Adolf Hitler, dictator of Nazi Germany at that time.

21.

Knowledge tested: U.S. History: Major historical events (e.g., slavery, Declaration of Independence, Civil War, Great Depression, etc.) 23. A. There was no armed conflict in California between abolitionists and slavery supporters in the 1850s. B. Correct Answer. Kansas experienced high levels of violence beginning in 1854 and continuing until 1861, due to the presence of large numbers of pro- and anti-slavery advocates fighting about the future of the territory. Under the doctrine of state sovereignty supported by Stephen Douglas, residents in the Kansas territory were struggling to determine whether Kansas would be admitted into the Union as a free state or a slave state; the resulting violence earned it the nickname Bleeding Kansas. It was eventually admitted as a free state in January 1861. C. There was no armed conflict in Oklahoma between abolitionists and slavery supporters in the 1850s. D. There was no armed conflict in Minnesota between abolitionists and slavery supporters in the 1850s.

Knowledge tested: World History and Geography: Understanding of key historical events A. Soviet leader Nikita Krushchev delivered his Secret Speech to a closed session of the 20th Party Congress in Moscow in February 1956. His speech attacked Stalins rule. B. The Communist Government of East Germany began building the Berlin Wall in 1961 to divide East and West Berlin. The Berlin Wall became the symbol of the Cold War until its destruction in 1989. C. Correct answer. At the same time as the Hungarian uprising in October 1956, a serious crisis occurred when the Egyptian government seized control of the Suez Canal. Because the Suez Canal provided access to Middle Eastern oil, Egypts actions were seen as a possible threat to cut off Europe's oil supply. The ensuing Suez Crisis threatened regional stability and caused Britain, France, Israel, the United States, the Soviet Union, and the United Nations to become involved. D. The first Boer War occurred in 1880-1881. The Second Boer War took place from 1899-1902. Both wars were fought against the British Empire.

25

24.

Knowledge tested: U.S. History: Events and trends in U.S. society (e.g., women and minority roles, social philosophy, etc.) A. Correct Answer. Title IX is a portion of the 1972 Education Amendments that covers equal treatment on the basis of sex in all educational activities, including athletics. The legislation states: No person in the United States shall, on the basis of sex, be excluded from participation in, be denied the benefits of, or be subjected to discrimination under any education program or activity receiving Federal financial assistance. Before Title IX, women constituted only 2% of all college athletes. According to 2010 statistics, women now constitute close to 45% of all college athletes at the NCAA Division 1 level. Nearly 3.5 million young women participate in high school sports, as compared with 4.4 million young men. B. Title IX legislation does not apply to elected offices held by any group. C. Title IX legislation does not apply to women in the armed services. D. Title IX legislation does not apply to employment of the disabled.

26.

Knowledge tested: Management Principles, Psychology, and Human Behavior: Knowledge of basic leadership principles, such as modeling A. Correct Answer. First and foremost, managers should set a good example for their employees. Managers set the tone for the workplace. By setting a good example of expected behaviors and performance, managers gain the respect of their employees. B. This sets a negative tone in the workplace. Positivity is much more effective, and a positive workplace is more productive than a workplace in which employees live in fear of reprimand. C. Expected behaviors should not be elevated to exceptional status. Employees should do whats expected without being rewarded or compensated. These rewards should be reserved for exceptional or above and beyond behavior. D. Monetary rewards for expected behavior also can be problematic because employees should not need extra incentives for simply displaying appropriate behaviors and doing their jobs. Sometimes monetary rewards can foster unhealthy competition among employees and hinder teamwork.

25.

Knowledge tested: Mathematics and Statistics: Basic knowledge of statistics (e.g., mean, mode, standard deviation, etc.) A. See B. B. Correct Answer. The new average salary is the old average of $30,000 plus the salary increase of $3,000 received by each employee (or $33,000). The standard deviation of a set of values doesnt change when the same quantity is simply added to each value, so the standard deviation is still $1,000. C. See B. D. See B.

27.

Knowledge tested: Computers and the Internet: Knowledge of basic computer functions and the Internet (World-Wide Web) A. No current technology processes data using light. B. The quantity of data storage is not addressed by cloud computing per se. C. Second Life is a game-like environment where persons can interact in ways that simulate human behavior. Cloud computing is best described as a service that supports data storage and computing applications. D. Correct Answer. Users access cloud computing through network devices, such as desktop computers, laptops, tablets, and smart phones. Cloud computing allows users to store data and to access computing applications as an online service (versus on local hardware).

26

28.

Knowledge tested: U.S. Government: Congress and its role in foreign affairs A. Recommital is a parliamentary procedure, usually afforded to the majority in a legislative chamber, giving opponents of a bill a final opportunity to send a measure back to committee after it has been passed by the chamber. B. Enrollment is the formal procedure for printing a bill in official, final format for presentation to the president for his signature or veto. C. Unanimous consent is a parliamentary procedure usually employed to facilitate the conduct of minor business and the passage of non-controversial measures. As the name suggests, unanimous consent requires the assent of all members. D. Correct Answer. In 1919, the Senate first invoked cloture (Senate rule XXII) to end a filibuster against the Treaty of Versailles with a two-thirds majority vote. In 1975, the Senate amended the rule, reducing the number of votes required to end a filibuster from two thirds to three fifths, or a supermajority of 60 votes. Cloture is the only formal procedure under Senate rules for ending a filibuster.

30.

Knowledge tested: World History and Geography: Geography-based national rivalries and alliances A. Correct answer. Kashmir is the site of territorial disputes with portions under the de facto administrative and military control of India (Jammu and Kashmir), China (Aksai Chin), and Pakistan (Azad Kashmir and GilgitBaltistan, formerly known as the Northern Areas). B. See A. C. See A. D. See A.

31.

Knowledge tested: U.S. economic issues A. Supply-side economic theory holds that higher tariffs would yield less profit to the market and less revenue to the government. Therefore, supply-side economics would not advocate for higher tariffs. B. Generally, a stock split changes the number and value of shares. It reduces a stocks share price, which may make it more affordable to small investors, but it is not intrinsically related to supply-side economic theory. C. Government subsidies typically are reduced or abolished in supply-side economic policy. Therefore, supply-side economics would not advocate for government subsidies. D. Correct answer. Supply-side economics advocates for lower marginal tax rates as an incentive to work, allow people to keep more of the money they earn, and boost economic growth.

29.

Knowledge tested: U.S. Government: The Constitution and its history A. The first amendment guarantees citizens the right to freedom of expression and assembly, prohibits the establishment of an official religion, and guarantees the right to free practice of religion. B. Correct Answer. The second amendment to the U.S. Constitution protects the right of the people to keep and bear arms. The second amendment version passed by Congress reads: A well regulated Militia, being necessary to the security of a free State, the right of the people to keep and bear Arms, shall not be infringed. C. The third amendment prohibits the quartering of troops in private homes during peacetime and permits it during wartime only in accordance with the law. D. The fourth amendment provides due process guarantees, including protection from unreasonable and warrantless searches and seizures of property.

32.

Knowledge tested: Communications: The medias influence A. See C. B. See C. C. Correct answer. The CNN effect is in reference to CNN as the first 24-hour news channel to shape public opinion with direct coverage of breaking events, which put pressure on policy makers. D. See C.

27

33.

Knowledge tested: Computers and the Internet: Knowledge of basic computer functions, E-mail functions, and the Internet (World-Wide Web) A. A computer virus is one of the most common computer threats and is most often found in e-mail attachments and downloads from malicious Web sites. It is a program that has the ability to copy itself to infect computers and cause such damage as corrupting computer files and even deleting everything on a hard drive. B. Rootkits employ a variety of surreptitious techniques to gain access to and control of a computer system for such malicious intent as stealing user passwords, credit card information, etc. Rootkits can be difficult to detect, but they do not replicate themselves on machines. C. Correct Answer. The big difference between a computer worm and a virus is that worms pose a threat even when computer users dont download any files. These programs use computer networks and the Internet to search for vulnerable computers. Worms replicate themselves to such a degree that they cause harm to the network. Computer users usually dont recognize that anything has happened until they begin to experience computer problems, such as a slow down in the loading of Web pages. D. A Trojan horse is an insidious computer threat that typically hides within other programs and can exist essentially anywhere. This program may disable antivirus software and remove firewalls, but it doesnt replicate itself or automatically spread to other computers.

34.

Knowledge tested: U.S. Government: Understanding of relationships and interactions among components of the federal government A. Standing committees are permanent committees that have legislative jurisdiction established by the U.S. House and Senate rules. They are committees in both chambers that have on-going memberships and jurisdictions, distinguished from ad hoc committees. B. Correct Answer. Conference Committees consist of a temporary, ad hoc panel of House and Senate members who have been appointed to resolve disputes and differences in bills passed. C. Authorizing committees are substantive standing committees that make decisions regarding public policy in particular issue areas and can report bills to the floor for final consideration in each chamber. Authorizing committees handle legislation that permits, but does not require, federal spending, and they set upper limits on the amount authorized for particular projects. D. Select committees are formed to handle special issues as they arise. They generally have a limited mandate, are often investigatory in nature, rarely have members from both chambers, and often do not have the authority to report legislation to the floor.

35.

Knowledge tested: Communications: Sources of information (e.g., newspapers, television, radio, published laws and governmental reports) A. USAID focuses on U.S. government aid to developing countries and those hit hard by health crises, war, or natural disastera focus incompatible with general information about all the worlds countries. B. This Bureau has recently discontinued its Background Notesa page of summary information on each country, never updated weeklyand replaced them with Country Fact Sheets that focus on U.S. relations with each country. C. OPIC concentrates on private investment as a form of development assistance to regions that have experienced instability or conflict, yet offer promising growth opportunities, such as to the Middle East and North Africa, SubSaharan Africa, and Southeast Asiaagain, incompatible with general information about all countries of the globe. D. Correct answer. The CIAs well-known World Factbook evolved from its mission to gather basic intelligence on all areas of the world.

28

36.

Knowledge tested: U.S. Government: National interests and objectives pursued abroad (e.g., human rights, terrorism, crime, promotion of U.S. trade, etc.) A. Correct Answer. The 1997 Kyoto Protocol to the United Nations Framework on Climate Change calls for stabilization of greenhouse gas concentrations in the atmosphere at a level that would prevent dangerous anthropogenic interference with the climate system. B. The Kyoto Protocol does not directly address ocean pollution. C. Trade in endangered species is controlled by the Convention on the International Trade of Endangered Species of Wild Flora and Fauna (CITES). D. The Kyoto Protocol does not deal with child labor, which is generally handled by the United Nations under its Convention on the Rights of the Child.

40.

Knowledge tested: Management Principles, Psychology, and Human Behavior: Understanding of basic strategies such as reward and punishment and motivational techniques (e.g., bonuses) A. Given that a monetary reward is involved, such an announcement might yield resentment, jealousy, or feelings of inadequacy among those team members who are not being recognized for their individual accomplishments. B. Correct Answer. This approach is likely to feel most special to the person receiving the award. The employees direct supervisor takes the time to recognize and reward the employee in person, in a timely manner, for the exemplary work performed. There also is no awkwardness for those team members who didnt receive the special monetary compensation. C. This scenario is less personal than a face-toface meeting, and a direct supervisor is more likely than an Ambassador to be familiar with the specific work for which the employee is being rewarded. D. A formal annual performance review is a separate and less timely event, and it is likely to contain constructive criticism as well as positive feedback. Because the monetary bonus is being given to the employee for exemplary work performed on a specific project, the employee should be informed of the reward soon after the work that was performed.

37.

Knowledge tested: World History: Historical events and their impact on foreign policy A. Correct answer. U.S. President Theodore Roosevelt invited diplomats from Russia and Japan to his home to negotiate an end to the Russo-Japanese War. Roosevelt won the Nobel Peace Prize for the negotiations that led to the Treaty of Portsmouth, which ended the war. B. See A. C. See A. D. See A. 41.

38.

Knowledge tested: World Geography: Geography-based national rivalries and alliances A. See B. B. Correct answer. Camp Lemonnier in Djibouti is the first official U.S. military base of operations for U.S. Africa Command in the Horn of Africa. C. See B. D. See B.

Knowledge tested: Communications: Public speaking techniques A. Giving audience members a chance to read the speech in advance might dilute the immediacy and effect of the speech, allowing the audience members attention to wander due to their advance familiarity with the presentation. B. Correct answer. Pauses and silence can be used to draw attention to different points made in the speech, have a dramatic effect, allow listeners to process what has been said, and build anticipation of what information is to come next. C. Highly technical language is not really meant to be spoken. It is comprehensible only to a narrow audience of specialists. D. The use of slang language might get attention, but it might attract the wrong kind of attention. There is the risk that slang could offend some audience members, which could undermine the primary intention of the speech.

39.

Knowledge tested: Mathematics: Fundamental principles of mathematics (e.g., multiplication, division; calculation of interest rates, etc.) A. B. C. D. Correct Answer. 55 x 4/5 + 70 x 1/5 = 58% See A. See A. See A.

29

42.

Knowledge tested: U.S. Government: Constitution (e.g., treaties) and its influence on relationships with other countries A. Correct Answer. The U.S. Constitution requires that two thirds of U.S. Senators approve of a resolution to ratify a treaty. While a majority of Senators in the Republican-controlled 66th Congress supported the Treaty of Versailles, disputes regarding reservations and amendments prevented the Senate from mustering the twothirds supermajority vote to approve a resolution to ratify it. Article II, section 2 of the U.S. Constitution provides that the president "shall have Power, by and with the Advice and Consent of the Senate, to make Treaties, provided two-thirds of the Senators present concur." B. The Treaty of Versailles established the League of Nations. C. The Treaty of Versailles (and associated treaties, such as the Treaty of Trianon) did, in fact, partition the Austro-Hungarian Empire, creating such new nation-states as Czechoslovakia and others, but this is not the reason that the Senate failed to ratify the treaty. D. President Wilson was a principal architect of the Treaty of Versailles and opposed efforts to amend it.

44.

Knowledge tested: U.S. Government: Congress and its role in foreign affairs A. A substitute measure keeps the original bill number and sometimes the bill title, but deletes the original content of the measure, replacing it with significantly different provisions. B. Exempting bills from most Senate rules of procedures generally requires invocation of a process termed unanimous consent. C. Correct Answer. Since the president has no line-item veto, non-germane provisions are often added to important or popular bills through the use of riders to ensure they will not be vetoed, as they would be if they were standalone bills. D. Recommital is the term used to refer to sending a bill back to committee for further deliberation.

45.

Knowledge tested: Management Principles, Psychology, and Human Behavior (e.g., delegation, supervision, and employee relations) A. For a project to be successful, each team member must work in his or her area of expertise to complete the project tasks, but it is essential that the team members have a common goal and recognize their dependence upon one another for the groups success. Each person should not perform as an independent contributor alone. B. See A and C. C. Correct Answer. To be successful, a project team must have a group vision and goal with clearly defined project responsibilities for each team member. Project tasks should be assigned based on each team members area of expertise and on the requirements for completion of the project. D. Project roles and responsibilities must be clearly defined, preferably with no overlap of accountabilities. Only one person should be accountable for each task assigned, although any number of people may contribute towards accomplishing different tasks for a project. Two or more people should never be accountable for the same task as this can lead to confusion and potential problems.

43.

Knowledge tested: Computers and the Internet: Knowledge of basic computer functions, E-mail functions, and the Internet (World-Wide Web) A. OCR does not identify and correct typographical errors in data files (see C and D below). B. OCR does not scan remote data into a database (see D below). C. OCR does not perform spell checks. Spell checkers flag misspelled words in an electronic document (e.g., in Word documents or e-mails) and can include specialized dictionaries that contain, for example, medical and foreign language terminology. D. Correct Answer. OCR is a scanning process in which a printed document is converted into a computerized text file, which can be manipulated for a variety of purposes. This technology has been used to create a computerized reading machine, which has the ability to read text out loud to individuals with visual impairments.

30

46.

Knowledge tested: U.S. History: Relationships between U.S. society and other countries A. Bilingual education received widespread support among immigrants to the United States after 1960. B. It would be difficult to characterize the growing immigrant populations in the United States as challenging affirmative action after 1960. C. Multiculturalism received widespread support by immigrants to the United States after 1960. D. Correct Answer. Assimilationism is the act of encouraging minority cultural groups to be similar to the majority culture. After 1960, immigrant groups in the United States challenged such practices as English language only educational policies in many states, pressuring school districts to hire teachers with multiple-language abilities in an effort to reverse assimilationism.

48.

Knowledge tested: Management Principles, Psychology, and Human Behavior: Understanding of basic strategies such as reward and punishment and motivational techniques (e.g., bonuses) A. This is an example of extrinsic motivation because all employees are being told specifically how to perform their job duties from an external sourcethe manager without regard for their individual interests, knowledge, and experiences. B. This is an example of extrinsic motivation because the manager is telling employees what tasks to perform and is delegating work based on availability alone, regardless of employees talents, interests, knowledge, and experiences. C. This is an example of extrinsic motivation because it is based on monetary rewards and for length of service only. D. Correct Answer. Intrinsic motivation refers to motivation that is driven by an interest or enjoyment in the task itself and comes from within the individual rather than from any external reward, such as money. This is the best response because the manager is creating a situation in which employees gain satisfying, valuable skills for their own sake. The focus is on learning skills that fit the employees interests as well as to promote job growth.

47.

Knowledge tested: Mathematics and Statistics: Basic knowledge of statistics (e.g., mean, mode, standard deviation, etc.) A. Correct Answer. The range is the highest value minus the lowest value. Before the increase in wage, the range was $10 $8 = $2. After the $1 increase in wage, the wages became $9, $10, and $11; so the range was $11 $9 = $2. $2 $2 = 0. Thus, there was no increase in the range. Increasing each employees wage by the same amount does not change the range. B. See A. C. See A. D. See A.

49.

Knowledge tested: U.S. Government: Political process and its impact on policy (e.g., role and philosophy of political parties, etc.) A. Libertarianism gives no special status to libraries. B. Libertarians oppose government constraints on individual behavior as inconsistent with personal liberty. C. A focus on community obligation is characteristic of the communitarian political perspective, not the libertarian perspective, which focuses on individual rights. D. Correct Answer. Libertarians stress personal liberty, including the right to accumulate and utilize private property with minimal government interference.

31

50.

Knowledge tested: U.S. Government: Specific aspects of the U.S. Constitution A. Correct Answer. Article I, section 8 of the U.S. Constitution lists the enumerated powers granted to the U.S. Congress, which include declaring war, minting money, regulating commerce, and collecting taxes. B. While manifest powers may be similar to enumerated powers in that both refer to readily obvious characteristics, this term is not generally used to refer to congressional powers. C. Executive powers refer to the authority granted to the President and associated Executive Branch offices and agencies. D. Congressional powers include both the enumerated powers explicitly listed in Article I, section 8 of the U.S. Constitution, as well as implied powers authorized broadly under the necessary and proper clause at the end of Article I, section 8.

52.

Knowledge tested: Management Principles, Psychology, and Human Behavior: Knowledge of basic leadership principles, such as modeling A. Correct Answer. Transformational leadership is about leading an organization through change. In its purest form, it is the ability to guide and direct employees within a given organization, focusing on one clear, directed vision. B. Transformational leadership is a key component of the U.S. Army as well as many large corporations. This leadership style is in no way limited to small workplaces with few departments and divisions. C. Transformational leadership is antithetical to bureaucratic structures. For example, inspirational motivation describes managers who motivate employees to commit to the vision of the organization. Managers with inspirational motivation encourage team spirit to reach goals of increased revenue and market growth for the organization. D. Vertical structures go against the transformational leadership principle known as individual consideration. Individual consideration describes managers who act as coaches and advisors to employees. Managers with individual consideration encourage employees to reach goals that help both the employees and the organization.

51.

Knowledge tested: Computers: Knowledge of basic computer and e-mail functions A. The To field of an e-mail lists the e-mail addresses for the recipients of an e-mail, which can be seen by anyone receiving the e-mail. B. The From field of an e-mail lists the e-mail address for the sender of the e-mail, which can be seen by anyone receiving the e-mail. C. The Cc field of an e-mail is used to send a copy of the same message to all e-mail addressees listed in the field, and their names can be seen by anyone receiving the e-mail. D. Correct Answer. The Bcc field of an e-mail is used to send a blind copy of the message to one or more recipients to maintain the confidentiality of each recipients identity. Bcc recipients are invisible to all other recipients of the message (including other Bcc recipients).

53.

Knowledge tested: U.S. History: Events and trends in U.S. society (e.g., women and minority roles, social philosophy, etc.) A. John Lockes best known work is Two Treatises of Government. B. Jean-Jacques Rousseaus best known works are Discourse on the Origin of Inequality and On the Social Contract. C. Correct Answer. American author Henry David Thoreau is best known for his book Walden, a reflection on the sanctity of living simply to transcend what he called overcivilization. Thoreaus essay Civil Disobedience has been credited as influencing Martin Luther King, Jr.s theory of nonviolent resistance. D. Thomas Hobbes best known work is Leviathan.

32

54.

Knowledge tested: Management Principles, Psychology, and Human Behavior: Feedback techniques A. Performance feedback should be constructive. Reprimanding an employee for unsatisfactory performance is often viewed as negative and punitive, and punishment rarely works as a long-term strategy to change employee performance and create a positive workplace environment. B. This approach is also negative and is focused primarily on the correction. Performance feedback is not primarily aimed at pointing out things an employee does wrong. C. Performance feedback pertains to a particular workplace setting and situation. It is not meant to be an assessment of an employees general abilities or limitations beyond the job setting. D. Correct Answer. Supporting an employees professional growth, development, and achievements is a constructive approach (rather than only accentuating the negative aspects of an employees performance). A supervisors goal should be to foster growth in employees, focusing on employees developmental goals and achievements in the workplace.

56.

Knowledge tested: U.S. Government: National interests and objectives pursued abroad (e.g., human rights, terrorism, crime, promotion of U.S. trade, etc.) A. Even if its common and accepted practice in a foreign country, this type of payment to a foreign official would violate the Foreign Corrupt Practices Act (FCPA) and would not be permissible. B. Even if not enforced in the host country, this type of payment would violate the FCPA and would not be permissible. C. Correct Answer. The FCPA makes it illegal for American businesses to offer anything of value to influence foreign officials in performance of their duties. D. U.S. officials should give relevant and accurate advice regarding furtherance of American business interests in accordance with local and American laws.

57.

Knowledge tested: U.S. History: Relationship between U.S. customs and culture on foreign policy A. Correct Answer. AGOA encouraged the development of new trade relationships between the United States and African countries through trade preferences (e.g., dutyfree exports of products from a sub-Saharan African textile and apparel industry to the United States). Rather than providing direct aid to Africa, the United States adopted this policy so that African economies could more directly benefit from the global demand for their products. B. This reverses the correct order of events. U.S. foreign policy had emphasized foreign assistance and educational uplift in Africa from the 1960s to the 1990s via the U.S. Agency for International Development (USAID), the Peace Corps, and other initiatives. C. While human rights became a hallmark of U.S. foreign policy under President Jimmy Carter in the 1970s, the primary U.S. policy toward Africa from the 1960s to the 1990s traditionally was aid and internal improvements. D. Prior to the mid-1990s, U.S. foreign policy emphasized aid to African countries rather than trade or human rights. An example of this earlier emphasis includes President Kennedys creation of the USAID in 1961, building upon the post-World War II legacy of the Marshall Plan in Europe.

55.

Knowledge tested: Computers and the Internet A. A spider software program tracks and indexes Web sites to provide up-to-date data for search engines. B. Correct Answer. A computer hacker can install and distribute a covert keylogger program via a Trojan horse or a virus to download data that has been logged onto that computer. C. Phishing is an attempt to obtain usernames, passwords, and other secure information by posing as an official entity in electronic communications. Recently, phishers have targeted banks, online payment services, and online IRS taxpayer sites. D. Broadcaster software can be used to broadcast webcam video feeds to the Internet.

33

58.

Knowledge tested: Management Principles, Psychology, and Human Behavior A. This response reflects the managers belief that host-country managers are valuable in key positions in the workplace, which is not indicative of an ethnocentric mindset. B. Correct answer. This mindset is ethnocentric because the manager values his or her own home countrys skills and knowledge over those of the host country. C. This response reflects the managers belief that host- and home-country managers are equally valuable, which is not indicative of an ethnocentric mindset. D. This belief is not ethnocentric because the home-country manager recognizes that hostcountry managers could have a better local management approach than the home-country managers.

60.

Knowledge tested: Communications: Public speaking techniques for particular target audiences A. Correct answer. A spatial presentation would be the most logical organizational pattern to use because it matches the medium to the message, which concerns orienting the audience spatially to the countrys geographical features. B. A categorical presentation would be too general. The discussion of different types of terrain would be best served by giving the audience a verbal map of the countrys specific location and geographical features. C. A problem-solution structure is unnecessary because the only problem in this scenario orienting the groupis best solved by giving them a verbal-guided tour of the countrys geographical features. D. A causal structure would be more appropriate for a scientific presentation than for an orientation describing a countrys geographical features.

59.

Knowledge tested: U.S. Government: Political system (e.g., separation of powers, etc.) A. The Legislative Branch, not the Executive Branch, has the power to remove judges through impeachment. B. Article III of the U.S. Constitution gives Congress, as a whole, the power to establish new federal courts, but not the President acting in cooperation with the Senate. C. Correct Answer. The Senate has sole power to confirm the Presidents judicial appointments. D. See C. The U.S. House of Representatives does not have the power to confirm the Presidents judicial appointments.

34

BIOGRAPHIC QUESTIONNAIRE TEST SECTION


The Biographic Questionnaire is a non-cognitive measure designed to assess an individuals experience. Past behavior has been shown to be highly related to job performance. Consequently, this type of questionnaire has been shown to be a good predictor of a variety of performance measures for many different jobs. The questions were written to assess relevant past interests, activities, and behaviors that are related to specific performance dimensions, such as communication and interpersonal conflict resolution skills. Unlike cognitive tests, this questionnaire does not consist of questions with right and wrong answers. Therefore, it is not a test for which advance preparation is necessary. Instead, the Biographic Questionnaire measures previous experiences that are related to successful job performance of Foreign Service Officers. The questions were written to assess relevant past interests, activities, and behaviors that are related to specific performance dimensions, such as communication, interpersonal, and problem-solving skills. When completing the Biographic Questionnaire, candidates should answer each item with their first impression. The best answer is an honest answer to the item. Therefore, candidates should not spend too much time answering any one item. Each dimension is measured by many different items written to assess a variety of contexts in which relevant behaviors may be exhibited. Items may assess the degree to which candidates have had particular experiences as well as their reactions to different situations. Some questions ask about candidates perceptions of others reactions to their behavior. The questions combine both subjective items as well as objective, highly verifiable items. On some questions, candidates might be asked to elaborate on their answer choice. For example, if a question asked how many jobs a candidate had held, he or she might be asked to list them. When completing their elaborations, candidates should be sure to manage their time effectively. Be as brief as possible. Since there is little space available for writing elaborations and the amount of time is limited, candidates may not be able to provide much detail. It is important for candidates to answer all of the questions in the Biographic Questionnaire to convey a complete picture of their previous experiences. Other examples of the types of items included in the Biographic Questionnaire are found below and on the next two pages. In developing the Biographic Questionnaire, items were subjected to multiple screenings to eliminate items that would be considered an invasion of privacy, not under the candidates control, and/or biased against race, gender, or national origin. Sometimes the relationship between an item and the dimension it measures is readily apparent. More frequently, though, the relationship is not as clear. This apparent lack of job-relatedness is a concern candidates sometimes express about biographic inventories. However, each of the items, whether apparent or not, was included on the questionnaire based upon specific relationships between the item and a performance dimension. Many of the biographic items are also multidimensional, which means that an item may measure several dimensions relevant to job performance. The Biographic Questionnaire measures a number of dimensions identified from job analyses as highly relevant for the successful job performance of Foreign Service Officers. The Biographic Questionnaire has been validated in accordance with professional standards and legal guidelines.

Sample Items for the Biographic Questionnaire


The Biographic Questionnaire section of the test consists of 77 questions administered in 42 minutes. There are 26 sample items provided below. Answer the 26 sample items by circling the choice that is most accurate and by elaborating on your choice where required.
1. During the last year, how often have you felt you had undertaken too many responsibilities? A. B. C. D. E. Very often Often Sometimes Seldom Never 35 2. When going out with friends, how often are you relied upon to decide where to go or what to do? A. B. C. D. E. Always Almost always Often Sometimes Rarely

3.

How many times over the last five years have you been in charge of planning an important big event (e.g., party, fund-raiser, wedding, reunion, important meeting)? A. B. C. D. E. Never Once Twice 3 or 4 times 5 or more times

10.

During the last two years, how many non-credit courses have you taken to improve your skills? A. B. C. D. E. 0 1 2 3 4 or more

11.

4.

If you answered B, C, D, or E to the previous question, briefly describe up to five of the events you planned. _________________________________________ _________________________________________ _________________________________________ _________________________________________ _________________________________________ In the past six months, how many gatherings have you attended where you knew very few people? A. B. C. D. E. 0 1 2 3 4 or more

If you answered B, C, D, or E to the previous question, list the names of the courses. _________________________________________ _________________________________________ _________________________________________ _________________________________________ _________________________________________ During the last six months, how often have you volunteered to help someone with a difficult task? A. B. C. D. E. Quite a few times Several times A couple of times Once Never

12.

5.

13.

6.

How difficult would your colleagues say that you find it to adjust to a change in your environment? A. B. C. D. E. Extremely difficult Very difficult Moderately difficult Somewhat difficult Not very difficult

If you answered A, B, C, or D to the previous question, briefly list the most significant tasks and how you helped. _________________________________________ _________________________________________ _________________________________________ _________________________________________ _________________________________________ In general, how likely has it been for you to mentally rehearse your thoughts before speaking? A. B. C. D. E. Very likely Likely Neutral neither likely nor unlikely Unlikely Very unlikely

14.

7.

How frequently have your friends or colleagues sought your help with resolving work- or schoolrelated problems? A. B. C. D. E. Very frequently Frequently Occasionally Rarely Never

15.

In general, how frequently have you engaged in hobbies or activities to reduce or minimize stress? A. B. C. D. E. Very frequently Frequently Sometimes Seldom Not at all

8.

How many of your past jobs or positions have required that you coordinate the activities of others? A. B. C. D. E. 0 1 2 3 4 or more 16.

How comfortable have members of other ethnic or cultural groups been when interacting with you? A. B. C. D. E.
Much more comfortable with me than with others Somewhat more comfortable with me than with others About as comfortable with me as with others Somewhat less comfortable with me than with others Much less comfortable with me than with others

9.

If you answered B, C, D, or E to the previous question, list the names of the positions. _________________________________________ _________________________________________ _________________________________________ _________________________________________ _________________________________________

36

17.

During the last six months, how often have you worked overtime to get something done? A. B. C. D. E. Very often Often Sometimes Seldom Very rarely

22.

During the last year, how often have you complimented someone for good service? A. B. C. D. E. Very often Often Sometimes Seldom Never

18.

If you answered A, B, C, or D to the previous question, list the approximate dates and how much overtime you worked each time. _________________________________________ _________________________________________ _________________________________________ _________________________________________ _________________________________________ How often would your classmates or coworkers say that you have complimented them for a good idea? A. B. C. D. E. Almost always Very often Often Sometimes Rarely

23.

How often have others described your ideas or solutions as creative or innovative? A. B. C. D. E. Very often Often Sometimes Seldom Never

19.

24.

If you answered A, B, C, or D to the previous question, briefly describe one or two of your ideas that were described as creative by others. _________________________________________ _________________________________________ _________________________________________ _________________________________________ _________________________________________ Compared to others, how comfortable are you in mediating conflicts between others? A. B. C. D. E. Much more comfortable than most people Somewhat more comfortable than most people About as comfortable as most people Somewhat less comfortable than most people A good bit less comfortable than most people

20.

How often do you make a grocery list before shopping? A. B. C. D. E. Almost always Often Sometimes Rarely Never

25.

21.

In school, how often did you take notes or create outlines to summarize book chapters or articles? A. B. C. D. E. Almost always Often Sometimes Rarely Never

26.

When presenting ideas to others, how frequently have you been asked to repeat yourself because you were not initially understood? A. B. C. D. E. Very frequently Frequently Occasionally Rarely Never

37

ENGLISH EXPRESSION TEST SECTION


The English Expression test section consists of 65 questions administered in 50 minutes. There are 70 sample questions provided below. Candidates should answer the 70 sample questions by circling the best answer choice for each item. After answering the sample questions, candidates can check their answers and review the rationales for the answer to each question in the next section of this Guide.

Sample Questions for the English Expression Test


DIRECTIONS: In the passages that follow, certain words and phrases are underlined and numbered. In the right-hand column, you will find alternatives for each underlined part. You are to choose the one that best expresses the idea, makes the statement appropriate for standard written English, or is worded most consistently with the style and tone of the passage as a whole. If you think the original version is best, choose option A., which is the same as the original version.
You also may find questions about a section of the passage, or about the passage as a whole. These questions do not refer to an underlined portion of the passage, but rather are identified by a number or numbers in a box. For each question, choose the alternative you consider best and circle the letter of that choice. Read each passage through once before you begin to answer the questions that accompany it. You cannot determine most answers without reading several sentences beyond the question. Be sure that you have read far enough ahead each time you choose an alternative.

Passage 1

1
World War II severely weakened Britain and France and hastened the end of the colonial era
1

1.

The writers main purpose for including the underlined portion is to: A. B. offer an opinion about the consequences of war. persuade the reader that Britain and France were weaker before World War II than after the war. introduce the main topic of this essay. highlight a difference between colonial African nations and their European oppressors. tending tended and tended which tended democracy and of resistance to aggression ideas democracy, and of resistance to aggressionideas democracy and of resistance to aggression; ideas democracy and of resistance to aggression. Ideas resonate on resonate to have resonated toward resonated with

in Africa. The war aims of the western powers

C. D.

2tending to undermine the ideological basis of colonialism by stressing the importance of 3democracy and of resistance to aggression ideas that resonate on Africas emerging
4

2.

A. B. C. D. A. B. C. D.

3.

nationalists. At President Roosevelts insistence, the United States and Britain agreed on a set of war
4. A. B. C. D.

38

5 aims in the 1941 Atlantic Charter that concurred the right of all peoples to choose the form of government under which they will live. The United Nations Charter signed at the end of the war committed the colonial powers to developing self 6 government and free political institutions in the

5.

A. B. C. D.

concurred testified contested affirmed

6.

territories under their control.

The writer is concerned about redundancy and wordiness and is considering deleting the underlined portion and ending the sentence with a period after the word institutions. Should the writer make this deletion? A. Yes, because the underlined portion repeats information that is already implied in Paragraph 1. Yes, because the underlined portion changes the focus of Paragraph 1 and does not provide a logical transition to the main topic of Paragraph 2. No, because the underlined portion provides a detail that clarifies the commitment made by the colonial powers under the UN Charter. No, because the underlined portion provides a reason for President Roosevelts insistence that the United States and Britain affirm the right of all nations to choose their own governments.

B.

C.

D.

2
With the coming of peace, Africans who had been educated abroad during the 1930s returned to Africa in the 1940s. Their thinking
7

7.

Which of the following choices provides the most informative context for the rest of Paragraph 2? A. B. C. D. in the 1940s. and began to organize opposition to the colonial regimes. after graduation. and to their families and home countries of origin. influenced had been an influence had been influenced having been influenced In addition, they They Moreover, they For example, they

8influenced by European intellectual currents of


9 the day. In addition, they had also studied the

8.

A. B. C. D. A. B. C. D.

9.

revolutionary movements sweeping Asia and the struggle of African Americans to achieve social justice. Some African nationalists were greatly
10 inspired by the work of W.E.B DuBois the

10.

African American sociologist and civil rights leader.

A. B. C. D.

DuBois the African American sociologist DuBois the African American sociologist, DuBois, the African American sociologist DuBois, the African American sociologist,

39

3
11.

By the end of the 1940s, British policymakers generally recognized that the African colonies would one day be independent; although they
11

A. B. C. D. A. B. C. D.

independent; although independent. Even though independent, nevertheless, independent, but by contrast, therefore, accordingly, for instance,

12.

tended to see that day as many decades in the


12 future. France, by contrast, was following a 13 policy of "assimilation" that in theory would

13.

The underlined portion primarily indicates that France: A. B. C. D. had less practical knowledge of Africa than Britain did. did not ultimately achieve its goals through its policy of assimilation. had more anti-colonialist sentiment than Britain did. did not initially believe that assimilation would lead to African independence. Africans, could be assimilated as French citizens Africans, could be assimilated as French citizens, Africans could be assimilated as French citizens, Africans could be assimilated as French citizens

have led to the integration of its colonies into a

14.

A. B. C. D.

greater France. Individual Africans, could be


14

assimilated as French citizens once they had achieved a certain standard of French education. All around Africa, however, nationalist sentiments that were mounting. 15 By the early 1950s, the
15.

Which change, if any, should be made to the preceding sentence? A. B. C. D. Remove the comma after the word however. Delete the word that. Change the word were to had been. Make no changes.

16.

British faced a guerrilla uprising in Kenya. 16 British decision-makers soon concluded that

Which of the following true statements would add the best supporting detail at this point in the paragraph? A. New political parties were staging anticolonial protests in Gold Coast (Ghana) and Nigeria. The colonial partition of Africa often had the effect of imposing artificial boundary divisions on Africa, separating peoples that had been linked by culture and trade. In most of southern Africa, independence was long delayed because of the presence of substantial white populations that generally opposed majority rule. One such French citizen, Leopold Senghora poet and philosopherlater became the President of Senegal.

B.

C.

D.

40

17their weakened country could ill afford the cost of suppressing what amounted to a rising
18

17.

A. B. C. D. A. B. C. D.

their weakened country could there weakened country would its weakened country could their weakened country can suppressing what amounted to a rising containing the suppression of the rise of suppressing a rise in an increasing and rising

18.

African nationalism, and the independence timetable was moved forward. 19


19.

During the editing process, the writer discovers that the following sentence has been left out of the essay: Ghana became independent in 1957, followed by Nigeria in 1960, Tanzania in 1961, and Kenya in 1963. If this sentence were added to the essay, it would most logically be placed: A. B. C. D. immediately after the first sentence in Paragraph 1. at the end of Paragraph 1. at the end of Paragraph 2. at the end of Paragraph 3.

__________________________________________________________________________________________________

Passage 2 1

In July 1956, Egyptian President Gamal


20.
20 Abdel Nasser seized control of the Suez Canal,

A. B. C. D.

Canal, and nationalized Canal and nationalized Canal, but he nationalized Canal, the result was the nationalization

and nationalized the British- and French-owned

company who managed them, threatening both


21

21.

A. B. C. D.

who managed them, which they managed, managing them that managed it,

British and French stock holdings and Europe's

41

access to Middle Eastern oil. The ensuing


22

22.

Which of the following would best replace the underlined word without changing the meaning of the sentence? A. B. C. D. subsequent activating issuing apparent

crisis had important consequences for U.S. relations with its European allies and the Middle East.

2
23Was the crisis anticipated? Previously, the United States and Britain had agreed to finance
23. Which sentence provides the best transition from Paragraph 1 and the best introduction to the information in Paragraph 2? A. B. C. D. Was the crisis anticipated? Unfortunately, such crises do happen. The crisis did not occur in a vacuum. Not surprisingly, a crisis transpired.

24 Egypts Aswan Dam, which was to be both a

24.

A. B. C. D.

symbol of modernization or a source of electricity.

which was to be both a symbol of modernization or both a symbol of modernization or which were a symbol of modernization and which was to be both a symbol of modernization and Accordingly, As a consequence, However, In other words,

25Accordingly, after Nasser made several moves that appeared friendly to the Communist bloc, the United States and Britain reneged on the agreement and withdrew financing. 26

25.

A. B. C. D.

26.

Which of the following sentences best concludes Paragraph 2 by providing a reason for Nassers actions, as they are presented in Paragraph 1? A. To subsidize the Aswan Dam, Nasser needed the revenue generated from tolls collected from ships using the Suez Canal. Because oil from the Middle East moved through the Suez Canal, Nasser had the power to threaten Europes oil supply. Prior to July 1956, Nasser had accepted that the company that managed the Suez Canal was owned by the British and French. Consequently, Nasser realized that the building of the Aswan Dam was important to Egypts future. it as a threat to its them as a threat to its it as a threat to their them as a threat to

3
[1] The United States regarded Nasser's nationalization of the Canal as a problem and

B.

C.

D.

pursued a diplomatic solution. [2] Britain and

27 France, however, viewed it as a threat to its

27.

A. B. C. D.

national interests. [3] They secretly contacted

42

the Israeli government and proposed that Israel


28

28.

A. B.

invade the Sinai Peninsula, and march toward the Suez Canal zone. [4] Then Britain and France would warn both Egypt and Israel to stay away
29.

C. D.

government and proposed that Israel invade the Sinai Peninsula, and march government and proposed that Israel invade the Sinai Peninsula and march government and proposed that Israel invade the Sinai Peninsula and marched government, and proposed that Israel invade the Sinai Peninsula marching in the zone on the pretext within the zone at the pretext into the zone at the pretense within the zone for the pretense warning, which they warning and warning and then they warning that

from the Suez Canal and land paratroopers in


29

A. B. C. D. A. B. C. D.

the zone on the pretext of protecting it. [5] In October 1956, Israeli forces crossed the border to defeat the Egyptian army in the Sinai. [6] Britain

30.

31.

and France then issued their

30

warning, which

The best placement for the underlined portion would be: A. B. C. D. where it is now. before the word Both to begin the sentence. after the word USSR. after the word events to end the sentence.

they landed troops as planned. [7] Both the United States and the USSR responded by demanding
31

32.

a cease-fire to these events. [8] In addition, the United States also called for the evacuation of Israeli, French, and British troops under the supervision of a special UN force. 32 [9] This force arrived in mid-November, and by the end
33

Which change, if any, should be made to Sentence 8? A. B. C. D. Make no change. Delete the phrase In addition and begin the sentence with the words The United States. Replace the word evacuation with the word abandonment. Insert the word administered before the word supervision.

33.

The writer most likely included the underlined portion to: A. B. C. D. explain why the UN special force arrived by mid-November. show that the U.S. demand for an evacuation had been partially met. suggest that the British and French no longer supported the Israelis. indicate that the United Nations had highly trained forces in 1956.

of the year, the last British and French troops had withdrawn. 34

34.

The writer plans to divide Paragraph 3 into two paragraphs. The new paragraph should begin with Sentence: A. B. C. D. 3. 4. 5. 6.

43

4
35Although the Suez conflict was a military defeat for Egypt, but Nassers status as the defender of Arab nationalism grew in the Arab world. The United States had improved its relations with Egypt, but fundamental disputes, between
36

35.

A. B. C.

D.

Although the Suez conflict was There is likely no doubt about the Suez conflict being Even if the Suez conflict had been The Suez conflict was

36.

A. B. C.

Israel and its neighbors, remained unresolved. 37

D.

Egypt, but fundamental disputes between Israel and its neighbors, Egypt, fundamental disputes, however, between Israel and its neighbors Egypt but fundamental disputes between Israel, and its neighbors, Egypt, but fundamental disputes between Israel and its neighbors

37.

The writer wants to conclude the article with a statement that foreshadows future events in the Middle East. Which of the following true sentences best meets the writers goal? A. When these disagreements resurfaced, the United States would again be drawn into conflict. Israel withdrew from Egyptian territory gained in the fighting but regained access to the Straits of Tiran. Relations between the United States and its British and French allies temporarily deteriorated in the months following the war. Britain and France lost influence in the region after the withdrawal of their troops from the Suez Canal zone.

B.

C.

D.

__________________________________________________________________________________________________ Passage 3

1
[1] In American slang, its usually not good if 38your number is up. [2] For 50 39years however, American diplomats

38.

Which of the following colloquial phrases fits best with the meaning of the sentence that follows? A. B. C. D. your number is up. someone has your number. its a numbers game. theres power in numbers. years however, years, however, years, however years however

39.

A. B. C. D.

44

40have found comfort in knowing that the Operations Center had their numberthe telephone number thats always manned and always answered by someone who is ready,
41

40.

A. B. C. D.

have found comfort in knowing find it comfortable to know finding comforting knowledge have found the knowledge of it to be a comfort

41.

Which of the following choices adds the most relevant detail to Paragraph 1? A. B. C. D. is ready, willing, and able to help. is trained and experienced in the position. can answer a question, find an expert, or help defuse a crisis. can address the call in a prompt and professional manner.

willing, and able to help. [3] Founded in 1961

42during Cold War emergencies, the Center

42.

Given that all of the following choices are true, which choice best explains why the Ops office was founded when it was? A. B. C. D. during Cold War emergencies, which is over a half-century ago, before the events of the late 60s and early 70s, when many of todays FSOs were not even born,

has grown from one officer, a cot, and a pitcher


43

43.

The most likely purpose for the underlined portion is to: A. emphasize that Ops staff are expected to work around the clock. contrast the bare-bones beginnings of the Center with what it is today. illustrate how emergencies often call for temporary shelter and water rationing. demonstrate that handling crises has become easier with the passage of time.

of water into a 60-person operation that consists of the 24-hour Watch and the Office of Crisis Management Support. 44
B. C. D.

44.

The writer discovers that the following sentence was left out of the first paragraph: The Watch connects Department principals with each other while it stays on top of breaking events. The best place to insert this sentence is: A. B. C. D. before Sentence 1, to begin the paragraph. after Sentence 1. before Sentence 3. after Sentence 3, to end the paragraph.

45

2
45. Ops is always just a phone call away. The 45 Secretary of State, speaking at the Centers 50th anniversary celebration, noted the story of a fighter pilot 46. A. B. C. D. speaking spoke has spoken having spoken

The underlined portion could also be placed after the word: A. B. C. D. out. before. plane. Libya. had participation have participated had participated participating pondered over just exactly what he ought to gave some serious consideration as to what he should thought over the options of what to wondered what to do, the Secretary asked? do, the Secretary asked? do? the Secretary asked. do? The Secretary asked.

who

46

earlier that year bailed out before his plane

crashed over Libya. A Libyan who

47

had participation

47.

in an International Visitor Program found the pilot and 48.


48

A. B. C. D. A. B. C. D.

pondered over just exactly what he ought to do with

49. him. So what did he


49

do, the Secretary asked?

A. B. C. D.

50. He called Ops.


50

Which of the following choices best fits the tone and style of the essay? A. B. C. D. A short time later, Long story short, It came to pass that It eventuated that

A short time later, an American

rescue team picked up the pilot and took him safely home, in large part because the IVP graduate remembered whom to call. 51 51.

If the writer were to add a third paragraph to this essay, what would it most likely contain? A. B. C. D. A description of the U.S. fighter pilots mission in Libya The mission statement of the International Visitor Program More examples of situations in which Ops resolved a problem Other sources of help that are available to people in crisis overseas

46

Passage 4

1
In August 2007, a team of 24 scientists and more than 70 U.S. Coast Guard crew members 52embarked Barrow, Alaska, with the Healy, the
53 largest Coast Guard icebreaker. Concentrating

52.

A. B. C. D.

embarked Barrow, Alaska, with withdrew from Barrow, Alaska, to go on board left Barrow, Alaska, in pursuit of an operation on departed Barrow, Alaska, aboard

53.

on their mission, scientists used a multibeam echo sounder to collect more than 10,000 kilometers of
54 bathymetric data on the Chukchi Borderland, a

A. B. C. D.

Concentrating on their mission, Over the following four weeks, Determined to gather pertinent information, In late summer with the sun below the horizon for three hours per day,

54.

A.

B.

large undersea continental shelf stretching more than 500 miles from the Alaskan shore. The
C.

D.

Chukchi Borderland, a large undersea continental shelf stretching more than 500 miles Chukchi Borderland, a large undersea continental shelf stretching more than 500 miles, Chukchi Borderland, a large undersea continental shelf, stretching more than 500 miles, Chukchi Borderland a large undersea continental shelf stretching more than 500 miles Department, this advanced Department that advances Department, it advanced Department and advanced

Healys mission involved the State Department,


55

55.

A. B. C. D.

this advanced the causes of science and


56.

diplomacy. 56

At this point, the writer is considering adding the following true sentence: As a Coast Guard cutter, the Healy is also a capable platform for supporting other missions in the Arctic, including search and rescue, ship escort, and environmental protection. Would this be a relevant addition to make here? A. B. Yes, because it informs the reader about the versatility of Coast Guard icebreakers. Yes, because it provides the reader with a historical reference for the scientific missions performed by the Coast Guard. No, because it distracts the reader by including information that is beyond the scope of this article. No, because it suggests that the Healy was not designed for the type of scientific research mentioned in this article.

C.

D.

47

2
The data collected were used to create threedimensional maps of the Arctic Ocean floor and 57revealed previous unknown underwater
57. A. B. C. D. revealed previous revealed previously previously revealed previously revealing

seamounts, glacial scours created while in a


58

58.

past ice age, and large craters thought to be formed by seeping gas. The Healy mission was also part of a larger effort to collect data to determine the
59

A. B. C. D.

while in a past during a passed earlier in a passed during a past

59.

limits of the extended continental shelf of the United States. Under the Law of the Sea

The writer is considering deleting the underlined portion and ending the sentence with a period after the word data. If this portion were deleted, the paragraph would lose primarily: A. B. C. D. evidence for a point made previously. an essential transition to the information that follows. the writers opinion about the relevance of the data collected. a digression that leads the reader away from the main point of the paragraph. is automatically granted 200 nautical miles of its are automatically granted 200 nautical miles of its is granted automatically 200 nautical miles of their was granted automatically 200 nautical miles of their However, Correspondingly, Unfortunately, As a result,

Convention, every coastal nation is


60

60.

A. B. C.

automatically granted 200 nautical miles of its continental shelf.

D.

61However, a nation is entitled to the part of its continental shelf that extends beyond 200 nautical miles if they have met a basic
62

61.

A. B. C. D.

62.

criteria. Nations have sovereign rights over this Extended Continental Shelf (ECS),

A. B. C. D.

they have met a basic criteria. it meets this criteria. it meets certain criteria. their criteria are met.

48

63which for the United States could cover an area of at least one million square kilometers.

63.

If the writer wishes to complete the sentence and conclude the paragraph by further defining a nation's sovereign rights over its ECS, which of the following true phrases best accomplishes this? A. which for the United States could cover an area of at least one million square kilometers. including over the exploration and exploitation of natural resources. which they would presumably exercise according to the dictates of the Convention. although in the case of Russia, the much publicized planting of its flag on the sea floor at the North Pole has no legal significance.

B. C. D.

3
More than two dozen countries, including
64

64.

A. B. C. D.

countries, including the United States, countries, including the United States countries including the United States, countries including the United States

the United States, are working to locate and


65

65.

identify their various continental shelves. Australia, Brazil, Ireland, Norway, and others already 66having finished analyzing data to compile their findings into submissions for the Commission on the Limits of the Continental Shelf, which was set up under the Convention. Delimiting the ECS of the United States will be a 10-year process
67

A. B. C. D.

locate and identify their various formulate their idiosyncratic define their respective explicitly dictate

66.

A. B. C. D.

having finished analyzing data to compile have finished analyzing data and have compiled finished analyzing data that compiled finishing the analysis of data and compiled

67.

A. B. C. D.

States will be States over States, is States,

requiring more missions to the Arctic and other


68

68.

areas where the United States has an ECS. The

If the underlined portion were deleted and the sentence were ended with a period after the word Arctic, the sentence would primarily lose: A. B. C. D. a comparison of the ECS of the United States to the ECS of other countries. an explanation of how countries in other areas of the world delimit their ECS. evidence that additional missions to the Arctic ECS are going to be essential. information about the scope of the project of delimiting the United States ECS.

49

Department is overseeing the multiagency ECS


69

69.

Which of the following choices would be LEAST acceptable to use as an alternative to the underlined word? A. B. C. D. leading dominating directing presiding over

Task Force. 70

70.

The writer is considering rearranging the paragraphs in this article, moving Paragraph 3 to the beginning, followed by Paragraphs 1 and 2. Should the writer reorganize the article in this way? A. Yes, because Paragraph 3 is the best way to introduce the main topic of this article, which is delimiting the extended continental shelf of the United States. Yes, because the information in Paragraph 1 would be easier to understand if the information in Paragraph 3 were presented first. No, because the information in Paragraph 3 regarding the extended continental shelf logically follows the information provided in Paragraph 2. No, because the best introductory paragraph for this article would be Paragraph 2, not Paragraph 3.

B.

C.

D.

50

Rationales for Sample English Expression Questions


Main sources used: The Elements of Style by William Strunk, Jr., and E. B. White; The Chicago Manual of Style, 16th edition.
1. Concept tested: Clearly state a purpose. A. The underlined portion does not express the writers opinion. Rather, it is factual and specific. B. The purpose of the underlined portion is not to persuade, and this choice describes the first part of the sentence, not the underlined portion. C. This is the best answer. The underlined portion effectively introduces the main topic of the essay: how the colonial era ended in Africa. D. The main purpose of the underlined portion is not to highlight a difference between African nations and their European oppressors. 2. Concept tested: Relate clauses (avoid sentence fragments). A. A complete sentence should contain a subject and a predicate. The verb tending creates a sentence fragment because it is not a finite verb and does not provide a predicate for the sentence. B. This is the best answer. This answer provides the finite verb (and past verb tense) required to form a complete sentence. C. The conjunction and is unnecessary and is used incorrectly in this context. D. This choice creates a sentence fragment by placing the verb tended in a dependent clause. 3. Concept tested: Punctuate relationships and sequences (avoid ambiguity). A. This is the best answer. This answer properly punctuates the final phrase. B. A comma should not split the paired parallel phrases (of democracy and of resistance). C. A semicolon is not the proper punctuation to precede this noun clause. A semicolon should be used to join two independent clauses. D. The period after aggression creates a sentence fragment after it, beginning with Ideas. 4. Concept tested: Use the idioms of standard written English. A. The context requires a past verb tense. Also, the preposition on is unidiomatic and does not fit the relevant sense of resonate. B. This choice is incorrect because of the verb tense (see A), and to is unidiomatic. C. This choice is incorrect because of the verb tense (see A). Also, toward is unidiomatic and does not fit the intended meaning. D. This is the best answer. The past verb tense (resonated) is correctly provided. Also, the preposition with is idiomatic and fits the intended meaning (the ideas appealed to the nationalists). 51 5. Concept tested: Choose words to fit meaning and function (use words precisely and fluently). A. Concurred does not fit the intended meaning. B. Testified does not fit the intended meaning. C. Contested is the opposite of the intended meaning. D. This is the best answer. Affirmed conveys the intended meaning: that the charter acknowledged political self-determination to be a right. 6. Concept tested: Judge the relevancy of material. A. The information cannot be inferred from the rest of Paragraph 1. B. The underlined section is consistent with the paragraphs focus on relations between colonial powers and African colonies, and it does provide a logical transition. C. This is the best answer. The underlined phrase specifies whose self government and institutions are in question. D. This choice reverses the order of events, suggesting that something that happened at the end of the war provides a reason for something that happened in 1941. 7. Concept tested: Make decisions about cohesion devices (openings, transitions, and closingsselect an effective statement relative to specific paragraphs). A. This choice provides information that is already implied by Paragraph 1. B. This is the best answer. This answer helps develop the paragraphs account of the growth of African nationalism. C. This choice provides information that does not contribute to the development of ideas in the paragraph. D. This choice provides information that does not contribute to the development of ideas in the paragraph, and the phrase home countries of origin is redundant.

8.

Concept tested: Form verbs (compound tenses). A. This choice lacks the direct object required by influenced, and the verb tense does not work properly with the prepositional phrase, creating an incomplete sentence. B. This choice lacks the proper verb tense and does not work properly with the prepositional phrase, creating an incomplete sentence. C. This is the best answer. This answer correctly forms the finite verb needed to complete the sentence, correctly introduces the prepositional phrase, and conveys the intended meaning. D. This choice creates a sentence fragment because it lacks a finite verb for the predicate.

12.

Concept tested: Establish logical order (choose the appropriate transition word or phrase). A. This is the best answer. The transitional phrase by contrast draws attention to the fundamental difference between British and French policies. B. This transitional word therefore incorrectly suggests that the French policy somehow follows logically from the British one. C. This choice does not fit the intended meaning. D. For instance does not introduce an example of the point made in the previous sentence. Rather, it introduces a contrasting idea.

13. 9. Concept tested: Edit and revise effectively (avoid redundancy). A. Use of the transitional phrase In addition is redundant with also in this sentence. B. This is the best answer. This answer eliminates the redundancy. C. Use of the transitional word Moreover is redundant with also in this sentence. D. Use of the transitional phrase For example does not introduce an example of the point made in the previous sentence but is used incorrectly to introduce a new idea. 10. Concept tested: Punctuate breaks in thought (parenthetical elementscomma, parentheses). A. The African American sociologist is nonrestrictive and should be introduced with a comma. B. The African American sociologist is nonrestrictive and should be introduced with a comma. Because the African American sociologist is coordinate with civil rights leader, it should not be followed by a comma. C. This is the best answer. Because the African American sociologist is non-restrictive, it is correctly punctuated with a comma after DuBois. However, no comma is required after sociologist because it is coordinate with civil rights leader. D. Because the African American sociologist is coordinate with civil rights leader, it should not be followed by a comma. 11. Concept tested: Relate clauses (avoid sentence fragments). A. Using a semicolon instead of a comma to separate independent and although creates a sentence fragment. B. Using a period instead of a comma to separate independent and Even though creates a sentence fragment. C. Using a comma instead of a period after independent creates a comma splice. D. This is the best answer. This answer uses a comma before the coordinating conjunction but to join two independent clauses.

Concept tested: Make decisions about appropriateness of expressions for audience and purpose. A. The underlined phrase does not imply a contrast between Britain and France. B. This is the best answer. The underlined phrase implies a contrast between what the theory predicted and what actually happened. C. See A. D. The underlined phrase does not imply any change in what France believed.

14.

Concept tested: Avoid unnecessary punctuation. A. A comma should not separate the subject of the sentence from its predicate. B. See A and C. C. The final clause of the sentence is restrictive and should not be introduced by a comma. D. This is the best answer. This answer does not have any unnecessary punctuation.

15.

Concept tested: Relate clauses (avoid sentence fragments). A. See C. Also, the parenthetical expression however should be set off with a pair of commas, not just one comma. B. This is the best answer. Removing that turns the fragment into a complete sentence. C. Changing the verb tense does not turn the sentence fragment into a complete sentence. D. This choice is not acceptable because that creates a sentence fragment.

16.

Concept tested: Make decisions about cohesion devices (openings, transitions, and closingsselect an effective statement relative to specific paragraphs). A. This is the best answer. This answer gives examples of mounting nationalist sentiments, the focus of this part of the paragraph. B. This choice does not help develop the idea that nationalist sentiments were mounting. C. This choice is contrary to the idea that nationalist sentiments were mounting. D. The preceding sentence does not mention French citizens. Thus, this choice does not fit the context, and it does not support the point of this part of the paragraph.

52

17.

Concept tested: Use pronouns (use the proper form of the possessives and distinguish them from adverbs [there] and contractions [its]). A. This is the best answer. The plural possessive pronoun their correctly refers back to the plural decision-makers, and the verb tense at the end is consistent with the context. B. See A. There is an adverb, not a pronoun. Also, the verb at the end does not fit the context. C. A plural pronoun is required instead of the singular possessive pronoun its to refer to the plural decision-makers. D. The present tense of the verb at the end is not consistent with the context.

21.

Concept tested: Assure grammatical agreement (pronoun with antecedent). A. The plural pronoun (them) does not refer back to a plural antecedent. B. The plural pronoun (they) does not refer back to a plural antecedent. C. See A. D. This is the best answer. The singular pronoun (it) correctly refers back to the singular antecedent (Canal).

22.

Concept tested: Choose words to fit meaning and function (use words precisely and fluently). A. This is the best answer. Subsequent has the same meaning as ensuing (i.e., coming after). B. This choice suggests a cause rather than a consequence. C. Issuing sounds similar to ensuing, but it does not have the same meaning. D. This choice changes the meaning of the sentence, which is about a real crisis, not an apparent one. Also, apparent does not fit the intended meaning of ensuing.

18.

Concept tested: Edit and revise effectively (avoid wordiness). A. The phrasing what amounted to is extraneous. It adds nothing significant to the meaning of the sentence. B. Containing does not make sense here, and the multiple prepositional phrases are wordy. C. This is the best answer. This answer conveys the intended meaning without wordiness. D. The phrase increasing and rising is repetitive, and an essential part of the meaning (suppressing) has been omitted.

23.

Concept tested: Make decisions about cohesion devices (openings, transitions, and closingsselect an effective statement relative to specific paragraphs). A. This choice does not fit the rest of Paragraph 2, which refers to prior events that contributed to the crisis, not if the crisis was foreseen. B. This choice does not fit the rest of Paragraph 2, which describes prior events that contributed to the Suez crisis in particular. C. This is the best answer. This answer refers back to the crisis mentioned in Paragraph 1 and introduces the prior events described in the rest of Paragraph 2. D. This choice merely repeats information already provided in the previous sentence.

19.

Concept tested: Make decisions about cohesion devices (openings, transitions, and closingsselect an effective statement relative to essay as a whole). A. The added sentence would disrupt the explanation in Paragraph 1 of how World War II hastened the end of the colonial era in Africa, and the sentence would be out of place in the chronological sequence. B. The added sentence would not provide a logical conclusion to the events described in Paragraph 1 and would be out of place in the chronological sequence of events. C. The added sentence is irrelevant to the focus on revolutionary ideas in Paragraph 2 and is out of place in the chronological sequence of events. D. This is the best answer. The events described in the added sentence occur after everything else in the passage and provide the logical conclusion to the developments described in Paragraph 3.

24.

Concept tested: Assure grammatical agreement (adjectives and adverbs with their corresponding nouns and verbs). A. Both does not agree logically or grammatically with or. B. See A. C. The plural verb (were) lacks grammatical agreement with the singular subject (Dam) in the clause. D. This is the best answer. This answer assures grammatical agreement, preserves the logical bothand construction, and correctly uses the singular verb (was) to refer back to the singular subject (Dam) of the clause.

20.

Concept tested: Avoid unnecessary punctuation. A. The verbs seized and nationalized have the same grammatical subject and should not be separated with a comma. B. This is the best answer. No comma is necessary before the coordinating conjunction and. Also, the intended meaning is conveyed. C. The coordinating conjunction but suggests a contrast, whereas seizing the canal prepares for nationalizing it, so there is no contrast. D. A semicolon (not a comma) should join two independent clauses. Use of a comma instead of a semicolon creates a comma splice. 53

25.

Concept tested: Establish logical order (choose the appropriate transition word or phrase). A. Accordingly suggests that the events described in this sentence logically follow from those in the previous sentence, but they are not in accordance with one another. B. See A. C. This is the best answer. However logically points to the contrast between the information in this sentence and the preceding sentence. D. In other words suggests that this sentence restates the information from the previous sentence, but the two sentences do not convey similar meanings.

29.

Concept tested: Choose words to fit meaning and function (avoid silly comparisons and expressions). A. This is the best answer. The prepositions in and on convey the intended meaning, which is that the paratroopers would land in the zone and that the main goal of Britain and France was not to protect the Suez Canal. B. The prepositions within and at do not convey the intended meaning: one does not land within the zone at the pretext. C. The word choices into, at, and pretense do not convey the intended meaning: one does not land into the zone at the pretense. D. The word choices within, for, and pretense do not convey the intended meaning: one does not land within the zone for the pretense.

26.

Concept tested: Make decisions about cohesion devices (openings, transitions, and closingsselect an effective statement relative to specific paragraphs). A. This is the best answer. This answer develops the ideas in Paragraph 2 by relating them to Paragraph 1, connecting the nationalization of the Suez Canal to Nassers need to pay for the Aswan Dam. B. This choice does not relate to the rest of Paragraph 2 and would be a poor conclusion for it. C. See B. D. This choice says nothing about financing for the Aswan Dam (the subject of Paragraph 2) and fails to relate the dam to the Suez Canal.

30.

Concept tested: Relate clauses (avoid faulty subordination and coordination). A. The relative pronoun which should not be used to join two independent clauses. B. This is the best answer. This answer uses no unnecessary words and preserves the parallel construction issuedand landed. C. The second use of then in this sentence is redundant, and they is unnecessary. D. This choice incorrectly makes warning the grammatical subject of landed, but the subject should be Britain and France.

31.

27.

Concept tested: Assure grammatical agreement (pronoun with antecedent). A. The singular pronoun its does not agree with the plural antecedent (Britain and France). B. See A. The plural pronoun them does not agree with the singular antecedent (nationalization). C. This is the best answer. The singular pronoun it agrees with the singular antecedent (nationalization), and the plural pronoun their correctly agrees with the plural antecedent (Britain and France). D. The plural pronoun them does not agree with the singular antecedent (nationalization). National interests requires a plural possessive pronoun to indicate whose national interests are in question.

Concept tested: Manage sentence elements effectively (use rhetorically effective subordination and combination). A. The adverbial phrase to these events should modify the verb responded, rather than ceasefire. Also, the phrase by demanding a ceasefire should be placed at the end of the sentence. B. The phrase by demanding a cease-fire is misplaced and ungrammatically separates the combined elements of the subject (Both and the United States and the USSR). C. The underlined phrase is misplaced and ungrammatically separates the subject from its predicate. D. This is the best answer. This answer correctly places the adverbial phrase to these events immediately after the verb it modifies (i.e., responded), and the phrase by demanding a cease-fire ends the sentence.

28.

Concept tested: Avoid unnecessary punctuation. A. A comma should not separate invade and march, because both verbs refer to the same grammatical subject (Israel). B. This is the best answer. This answer uses the correct verb forms to convey the intended meaning, without unnecessary punctuation. C. This choice suggests that Britain and France proposedand marched, but the context makes clear that only Israel does the marching. D. The verbs contacted and proposed have the same grammatical subject (They) and should not be separated by a comma. Marching in this context incorrectly conveys that Peninsula is doing the marching. 54

32.

Concept tested: Edit and revise effectively (avoid redundancy). A. This choice does not revise the sentence to eliminate the redundancy of in addition with also. B. This is the best answer. The redundant use of in addition with also has been eliminated. C. See A. The proposed change also fails to convey the intended meaning, since evacuation means removing the troops and abandonment involves leaving them in place. D. See A. The proposed change is extraneous, adding nothing significant to the meaning.

33.

Concept tested: Clearly state a purpose. A. The underlined portion gives no information about the arrival of the UN special force. B. This is the best answer. British and French troops had withdrawn, but it can be inferred that Israeli troops remained in Egypt even though the United States had called for them to leave as well. C. This choice makes too broad a generalization based on very limited information about British and French relations with the Israelis. Nothing in the passage necessarily indicates that relations had cooled. D. The underlined portion provides no information about the role of the UN troops in the withdrawal.

36.

Concept tested: Avoid unnecessary punctuation. A. There should not be a comma after neighbors, which separates the subject (disputes) from the predicate of the clause (remained unresolved). B. Removing the coordinating conjunction (but) creates a comma splice. C. The wording and its neighbors is not parenthetical and should not be set off by a pair of commas. D. This is the best answer. This answer properly joins the two independent clauses with a comma before the coordinating conjunction (but), without adding unnecessary punctuation.

37.

Concept tested: Make decisions about cohesion devices (openings, transitions, and closingsselect an effective statement relative to essay as a whole). A. This is the best answer. This answer hints at future events without revealing them. B. This choice concerns the situation at the end of the crisis, not future events. C. This choice says nothing about future events in the Middle East. D. This choice states something that happened after the crisis but does not hint at future events.

34.

Concept tested: Establish logical order (order paragraphs in a logical sequence). A. Sentences 2 and 3 are closely connected, describing how Britain and France responded to nationalization of the Suez Canal, so to add a paragraph break here would disrupt the narrative. B. Sentences 4 and 5 are closely connected, describing the planned invasion, so to add a paragraph break here would disrupt the narrative. C. This is the best answer. This answer divides Paragraph 3 in a logical way because earlier sentences describe the planning of the invasion, and Sentence 5 begins the account of the invasion itself. D. Sentences 5 and 6 are closely connected, describing fulfillment of the plan, so to add a paragraph break here would disrupt the narrative.

38.

Concept tested: Make decisions about appropriateness of expressions for audience and purpose. A. Although this expression refers to something that is not good, it does not fit the meaning established in the next sentence. B. This is the best answer. This answer refers to something that is usually not good, and it best establishes the play on words continued with had their number in the next sentence. C. This expression refers to something that may not be good but does not fit the meaning established in the next sentence. D. This expression refers to something that is usually good and does not fit the meaning established in the next sentence.

35.

Concept tested: Relate clauses (avoid faulty subordination and coordination). A. Although makes the first clause dependent, but it is joined to the rest of the sentence as an independent clause. B. This choice is wordy. C. Even if makes the first clause dependent, but it is joined to the rest of the sentence as an independent clause. D. This is the best answer. This answer avoids wordiness and opens the sentence with the independent clause it needs.

39.

Concept tested: Punctuate breaks in thought (parenthetical elementscomma, parentheses). A. This choice lacks one of the necessary commas (after years) to set off however. B. This is the best answer. As a parenthetical expression, however should be set off with a pair of commas. C. See A. D. This choice lacks the pair of commas required to set off however.

55

40.

Concept tested: Avoid unnecessary shifts in construction (person, number, voice, tense, mood). A. This is the best answer. This answer correctly provides the predicate the sentence needsa finite verb in the present perfect tense, which fits the intended meaning of the sentence (i.e., that the comfort the diplomats found in the past continues into the present). B. The verb is in the simple present tense, which does not fit the time established by the opening phrase of the sentence (the past 50 years). C. This choice does not provide the finite verb needed for the predicate of the main clause and creates an incomplete sentence. D. This choice is wordy, introduces a pronoun without a proper antecedent, and does not match the grammar of the rest of the sentence.

44.

Concept tested: Establish logical order (order sentences or parts of a sentence in a logical way). A. This choice provides a weak introduction to the paragraph by focusing on a subject that is not mentioned again until Sentence 3. B. This choice interrupts the pair of sentences about having someones number, which should be kept together. C. This choice interrupts the general description of the Center with a description of one of its parts that is not introduced until the next sentence. D. This is the best answer. This answer uses the inserted sentence to explain something mentioned in the preceding sentence.

45.

Concept tested: Relate clauses (avoid faulty subordination and coordination). A. This is the best answer. This answer has the proper verb form for the subordinate clause and is punctuated correctly. B. The sentence is punctuated incorrectly with the use of the past verb tense spoke. There should not be a comma between the subject and predicate of a clause, and the first clause is awkwardly joined to the verb phrase that begins with noted. C. See B. The present perfect verb tense does not match that of the following verb phrase. D. The verb tense does not fit the sentence because the main clause describes what the Secretary said, not what the Secretary did afterward.

41.

Concept tested: Judge the relevancy of material. A. This choice simply says the person is helpful, without providing any details about how he or she helps. B. This choice simply indicates that the person knows his or her job without providing any details about it. C. This is the best answer. This answer does not simply assert that the person is helpful but gives specific examples of how the person can help. D. This choice simply describes the way the person performs his or her job without providing any details about it.

42.

Concept tested: Make decisions about appropriateness of expressions for audience and purpose. A. This is the best answer. This answer relates the date of the founding to historical concerns. B. This choice provides no new information at all and explains nothing. C. See B. D. The information provided by this choice does not explain why the office was founded in 1961.

46.

Concept tested: Use modifiers (place modifiers so that they modify the appropriate element). A. This choice separates the verb bailed out from the phrases that explain it. B. This choice interrupts the prepositional phrase before his plane. C. This choice interrupts the subject and predicate (plane crashed) of the final clause. D. This is the best answer. Putting the underlined portion at the end of the sentence allows it to modify the whole description of what the pilot did without interrupting that description.

43.

Concept tested: Clearly state a purpose. A. The underlined portion does convey this information, but doing so is not its main purpose. B. This is the best answer. The sentence indicates that the office has grown from a humble beginning into an impressive operation, and the details in the underlined portion establish the first part of that pattern. C. The underlined portion does not illustrate the point stated by this choice. D. The underlined portion does not demonstrate the point stated by this choice but instead suggests just the opposite. 47.

Concept tested: Use the idioms of standard written English. A. Had participation is unidiomatic and does not fit the intended meaning of the clause. B. This choice provides a plural verb, but the subject of the clause is singular (A Libyan). C. This is the best answer. This answer provides the finite singular past perfect verb that the clause requires. D. This choice does not provide the finite verb that the clause requires.

56

48.

Concept tested: Edit and revise effectively (avoid wordiness). A. This choice uses more words than necessary to express the intended meaning. B. See A. C. See A. D. This is the best answer. This answer expresses the intended meaning clearly and succinctly.

53.

Concept tested: Make decisions about cohesion devices (openings, transitions, and closingsselect an effective statement relative to specific paragraphs). A. This choice says something that readers should take for granted and adds no significant information. B. This is the best answer. This answer adds significant information about the mission. C. See A. D. This choice adds extraneous, insignificant details about the mission.

49.

Concept tested: Punctuate breaks in thought (end of sentenceperiod, exclamation point, question mark). A. This choice punctuates the whole sentence as if it were a question, which it is not. B. This choice punctuates the whole sentence as if it were a quotation, which it is not. C. This is the best answer. This answer treats the quoted sentence as a question and keeps it with the clause indicating who said it. D. This choice punctuates the two clauses as if they were meant to be separate sentences, which they are not.

54.

Concept tested: Avoid unnecessary punctuation. A. This is the best answer. This answer correctly introduces the appositive phrase with a comma, with no unnecessary punctuation. B. This choice contains an unnecessary comma at the end, which separates the final phrase from the one it modifies. C. Stretching more than 500 miles is incorrectly set off by commas as if it were a parenthetical phrase, but it is not a parenthetical phrase. D. The long appositive after Borderland is not introduced with the necessary comma.

50.

Concept tested: Make decisions about appropriateness of expressions for audience and purpose. A. This is the best answer. This expression conveys the intended meaning in a simple, direct manner that fits the tone and style of the rest of the passage. B. This expression uses slang language that is not appropriate for the context. C. This expression is too formal for the context. D. This expression is too stilted for the context.

55.

Concept tested: Relate clauses (avoid comma splices). A. This choice creates a comma splice. B. This choice does not convey the intended meaning, but modifies State Department with a clause that does not apply to it. C. See A. D. This is the best answer. This answer correctly uses a coordinating conjunction to combine the two verb phrases.

51.

Concept tested: Judge the relevancy of material. 56. A. This choice shifts the focus of the essay from the Ops Center to the pilot, who was simply mentioned as a passing example. B. This choice shifts the focus of the essay from the Ops Center to the International Visitors Program, which was simply mentioned in passing. C. This is the best answer. This answer maintains the focus on the Ops Center. D. This choice shifts the focus of the essay from the Ops Center to a more general topic. 57. Concept tested: Judge the relevancy of material. A. The new information is not sufficiently relevant to the focus of the passage. B. The new information is not historical and not sufficiently relevant to the focus of the passage. C. This is the best answer. The new information is not sufficiently relevant to the focus of the passage. D. The new information lists some of the Healys capabilities but does not rule out others. Concept tested: Assure grammatical agreement (adjectives and adverbs with their corresponding nouns and verbs). A. Use of the adjective previous fails to convey the intended meaning (i.e., that the data reveal existing underwater features). B. This is the best answer. This answer accurately conveys the intended meaning and the adverb previously correctly corresponds to the past verb tense revealed. C. Since the data were newly gathered, they could not have revealed anything previously. D. See C. Revealing is not parallel with the other main verb (were used). 57

52.

Concept tested: Choose words to fit meaning and function (use words precisely and fluently). A. Embarked does not take the name of a place for its object, and people embark on a ship, not with it. B. Withdrew suggests retreating, which is contrary to the intended meaning of going forth, and to go on board is wordy and inaccurate. C. In pursuit of an operation is wordy, adding nothing to our understanding of what the scientists and crew are doing. D. This is the best answer. The intended meaning is conveyed accurately and succinctly.

58.

Concept tested: Use the idioms of standard written English. A. This choice indicates that the scours already existed when they were created, which does not make sense. B. This choice incorrectly uses a homonym of past, which does not convey the intended meaning. C. See B. D. This is the best answer. This answer completes the prepositional phrase so that it accurately conveys the information about when the scours were created.

62.

Concept tested: Assure grammatical agreement (pronoun with antecedent). A. The plural pronoun (they) refers back to miles, but in the context of this passage, it does not make sense to speak of miles as meeting criteria. B. This suggests that a criterion has been stated, but it has not, and the singular this does not agree in number with the plural criteria. C. This is the best answer. The singular pronoun it agrees in number with its singular antecedent (nation) and conveys the intended meaning. D. The plural possessive pronoun (their) refers back to miles, but in the context of this passage, it does not make sense to speak of miles as having criteria.

59.

Concept tested: Make decisions about appropriateness of expressions for audience and purpose. A. The information in the underlined portion has not been mentioned previously. B. This is the best answer. The underlined portion effectively introduces the topic of why data about the extended continental shelf are important. C. The underlined portion does not express the writers opinion. D. The underlined portion is not a digression from the main point of the paragraph but introduces an important aspect of it.

63.

Concept tested: Make decisions about cohesion devices (openings, transitions, and closingsselect an effective statement relative to specific paragraphs). A. This choice does not talk about a nations sovereign rights. Thus, it does not define them further. B. This is the best answer. This answer specifies the nature of certain sovereign rights. C. This choice talks about a nations sovereign rights but does not define them further. D. This choice does not talk about a nations sovereign rights. Thus, it does not define them further.

60.

Concept tested: Assure grammatical agreement (predicate with subjects of varying complexity including compound subjects, collective nouns). A. This is the best answer. The predicate and singular possessive pronoun its grammatically agree with the singular noun (nation). B. The plural verb are does not grammatically agree with the singular subject (nation). C. The plural possessive pronoun their does not grammatically agree with its singular antecedent (nation). D. See C.

64.

Concept tested: Punctuate relationships and sequences (simple phrases and clauses in a series). A. This is the best answer. A pair of commas correctly sets off the parenthetical phrase (including the United States). B. A pair of commas should enclose the parenthetical phrase (including the United States). C. See A and B. D. See A and B.

61.

Concept tested: Establish logical order (choose the appropriate transition word or phrase). A. This is the best answer. However establishes a contrast between the information in this sentence and in the preceding sentence. B. Correspondingly suggests that this sentence agrees more fully with the preceding sentence than is the case. C. Unfortunately puts the information in this sentence in a negative light that is not justified by the passage. D. As a result suggests that the statement in this sentence follows as a consequence of the previous sentence, which is not the case.

65.

Concept tested: Choose words to fit meaning and function (avoid silly comparisons and expressions). A. The verbs locate and identify are wordy and do not fit the intended meaning, and various is vague and inappropriate, since the passage implies that each nation has one shelf. B. Continental shelves are natural formations and hence the verb formulated does not convey the appropriate meaning. Also, the adjective idiosyncratic is not the proper word choice for defining the shelves distinctiveness. C. This is the best answer. The word choices are idiomatic and accurately convey the intended meaning. D. Continental shelves are natural formations; hence, the verb dictate does not fit the intended meaning, and explicitly dictate is wordy.

58

66.

Concept tested: Relate clauses (avoid sentence fragments). A. This choice creates a sentence fragment because it has no finite verb to form the predicate. B. This is the best answer. This answer provides the compound predicate required to form a complete sentence. C. Data cannot be said to compile findings. D. This choice does not provide the necessary parallelism between the two verbs in the compound predicate.

69.

Concept tested: Choose words to fit meaning and function (maintain the level of style and tone). A. Like overseeing, the word leading suggests a kind of supervision and guidance consistent with the idea of multiagency cooperation. B. This is the best answer. Dominating has overtones of supremacy and forceful imposition, which are inconsistent with the idea of multiagency cooperation. C. See A. D. See A.

70. 67. Concept tested: Relate clauses (avoid sentence fragments). A. This is the best answer. This answer provides the predicate required to form a complete sentence, with no unnecessary punctuation. B. This choice creates a sentence fragment because it has no finite verb to form the predicate. C. The grammatical subject should not be separated from the predicate with a comma. D. See B. 68. Concept tested: Clearly state a purpose. A. The underlined portion concerns only the ECS of the United States. B. See B. C. The underlined portion concerns only ECS outside the Arctic. D. This is the best answer. The passage has been focused on the Arctic, but the underlined portion is a reminder that the United States ECS extends into other areas.

Concept tested: Establish logical order (order paragraphs in a logical sequence). A. The topic of delimiting the ECS of the United States is better introduced by Paragraph 2, which explains ECS and the international convention governing it. B. The information in Paragraph 3 does not clarify information in Paragraph 1 and would be harder to understand if it came first. C. This is the best answer. Paragraph 2 introduces the topic of delimiting the ECS and claiming rights over it. Paragraph 3 develops the topic further by describing current efforts of various nations, particularly the United States. D. Paragraph 1 is the best introductory paragraph, explaining the origin of the data discussed in Paragraph 2.

59

SAMPLE WRITTEN ESSAY TOPICS


The Written Essay is used to evaluate each candidates ability to analyze a substantive topic, organize and develop ideas, and express them in correct and readable English prose. Candidates will be asked to respond to a topic that is randomly assigned. Representative topics include U.S. and/or international social systems and issues, customs and culture, history, education, religion, employment issues, etc. Essays are evaluated using established scoring criteria, such as the ability to analyze a topic, clarity of purpose, sentence structure, grammar, and mechanics, but not on the opinions expressed. When composing their essays, candidates should present and support their points of view clearly. The Written Essay will be evaluated on the quality of the writing, not on the opinions expressed. A successful essay should have an obvious structure and clear thesis supported by relevant substantiating details. It should show the candidates ability to analyze a topic in a way that is appropriate for the intended audience. The writing should be coherent with only occasional lapses that do not impede the organizational flow or the readers comprehension. Language should be generally concise with clear and appropriate word choices. The language also should be free of errors in grammar and syntax, with only minor errors in spelling and punctuation. There is no limit on length. Candidates will have 30 minutes to write an essay on the assigned topic on the exam. Two sample topics are provided below.

1. The United States has had a longstanding debate about public policy regarding access to healthcare for Americans. Some think that the current system in which individuals obtain their own healthcare coverage, either through an employer or by paying for it themselves, is adequate. Others think the federal government should underwrite health insurance coverage for all U.S. citizens, but that people should freely choose their healthcare providers. Still others think the United States should develop a national healthcare system of publicly funded doctors, clinics, and hospitals that would ensure access to healthcare for all Americans. In your view, how should access to healthcare be provided in the United States? Carefully explain the rationale for your position. 2. School boards and educators in the United States have begun to implement measures to ensure student safety as school violence seems to become more prevalent. Some schools have installed metal detectors and instituted locker searches to ensure there are no weapons on school property. Other schools have addressed personal expression seen as problematic by banning some types of clothing and hairstyles. However, many parents and students argue that these measures infringe on students rights and privacy and may create situations in which students are misunderstood or even harassed. In your view, where should the line be drawn between student safety and student privacy rights? Carefully explain the rationale for your position.

60

SUGGESTED STUDY MATERIALS


The Foreign Service Officer Test (FSOT) measures broad knowledge in world and national affairs, usually gathered over an extended period of time through education, reading, and life experiences. The best preparation is a good education, ideally one that includes courses in U.S. and world history, U.S. government and politics, international relations, political science, international economics and trade, geography, literature, English, management, and public administration. Familiarity with American society and culture is important. Because the test covers such a broad range of topics, it is difficult to provide a formal, comprehensive reading list that will prepare someone to pass the test. Below, we suggest an illustrative list of the types of books and other readings that would prove useful.

Current Affairs
U.S. News and World Report, The Economist A major daily newspaper such as The Wall Street Journal, New York Times, Washington Post, Los Angeles Times Journals such as Foreign Affairs and Foreign Policy

English Language Usage


Strunk, W., Jr. and White, E.B. The Elements of Style, 4th ed. Boston: Allyn & Bacon, 1999. ISBN 020530902X. Chicago Manual of Style: The Essential Guide for Writers, Editors, and Publishers, 16th ed. Chicago: University of Chicago Press, 2010. ISBN 0226104206.

United States (culture, foreign policy, history, politics)


Davidson, J.W., et al. Nation of Nations: A Narrative History of the American Republic, 5th ed. Boston: McGraw-Hill, 2004. ISBN 0072870982. Feagin, J.R. and Feagin, C.B. Racial and Ethnic Relations. 9th ed. Upper Saddle River, NJ: Prentice Hall, 2011. ISBN 9780205024995. Goldstein, J.S.and Pevehouse, J.S. International Relations, 10th ed. New York : Pearson Longman, 2011. ISBN 9780205059577. Hirsch E.D, Kett, J.F., and Trefil, J. The New Dictionary of Cultural Literacy. 3rd ed. Boston: Houghton-Mifflin, 2002. ISBN 0618226478. Norton, M.B., et al. A People and a Nation: A History of the United States, 9th ed. Boston: Houghton Mifflin, 2011. ISBN 9780495915256. Rosati, J. The Politics of United States Foreign Policy. 5th ed. Belmont, CA : Wadsworth/Thomson Learning, 2010. ISBN 9780495797241. Woloch, N. Women and the American Experience, 5th ed. Boston: McGraw-Hill, 2011. ISBN 0073385573.

World History and Geography


Atlas of the World, 18th ed. New York: Oxford University Press, 2011. ISBN 9780199829958. Craig, A.M., et al. The Heritage of World Civilizations, 9th ed., combined vol. Upper Saddle River, NJ: Pearson Prentice Hall, 2011. ISBN 9780205803507. Rumer, B. and Zhukov, S. Central Asia: The Challenges of Independence. Armonk, NY: M. E. Sharpe, 1998. ISBN 0765602547. Skidmore, T.E., Green J., and Smith, P. Modern Latin America. 7th ed. New York: Oxford University Press, 2010. ISBN 9780195375701. 61

Economics and Public Policy


Hall, R.E. and Papell, D.H. Macroeconomics: economic growth, fluctuations, and policy. 6th ed. New York: W.W. Norton, 2005. ISBN 0393975150. Mankiw, G. Principles of Microeconomics, 6th ed. Mason, OH : Thomson South-Western, 2012. ISBN 0538453044. Rushefsky, M.E. Public Policy in the United States: At the Dawn of the Twenty-First Century, 4th ed. Armonk, N.Y.: M.E. Sharpe, 2008. ISBN 9780765616630. Shultz, G.P. and Dam, K.W. Economic Policy Beyond the Headlines, 2nd ed. Chicago: University of Chicago Press, 1998. ISBN: 0226755991.

Management and Human Behavior


Gerrig, R.J. Psychology and Life, 20th ed. Boston : Pearson, 2013. ISBN 0205859135. Gleitman, H., Gross, J., and Reisberg, D. Psychology, 8th ed., New York: W.W. Norton & Co., 2011. ISBN 0393932508 (hc). Griffin, R.W. Fundamentals of Management, 6th ed. Mason, OH: South-Western Cengage Learning, 2012. ISBN 0538478756. Moorhead, G. and Griffin, R.W. Organizational Behavior: Managing People and Organizations, 10th ed. Mason, OH: South-Western/Cengage Learning, 2012. ISBN 0538478136. Schneider, S., and Barsoux, J., Managing Across Cultures, 2nd ed. Harlow, England; New York : Financial Times Prentice Hall, 2003. ISBN 027364663X. Twomey, D.P. Employment Discrimination Law: A Managers Guide : Text & Cases. 6th ed. Mason, OH : Thomson/SouthWestern, 2005. ISBN 0324271301.

Communication and the Media


Dominick, J.R. The Dynamics of Mass Communication: Media in Transition, 11th ed. Boston: McGraw-Hill, 2010. ISBN 0073378887. Houston, B., (eds.). The investigative reporters handbook: a guide to documents, databases, and techniques, 5th ed. Boston: Bedford/St. Martins, 2009. ISBN 0312589972. Itule, B.D. and Anderson, D.A., News Writing and Reporting for Todays Media, 7th ed. Boston: McGraw-Hill, 2006. ISBN 0072981091. Morrison, T., and Conaway, W. A. Kiss, Bow, or Shake Hands. 2nd ed. Avon, MA: Adams Media, 2006. ISBN 1593373686. Osborn, M., and Osborn, S. Public Speaking: Finding Your Voice Plus NEW MyCommunicationLab with eText, 9th ed. Boston: Houghton Mifflin, 2012. ISBN 0205912257. Samovar, L.A. Porter R.E., and McDaniel, E.R. eds. Intercultural Communication: A reader, 13th ed. Belmont, CA: Thomson/Wadsworth, 2011. ISBN 0495898317.

Computer Applications
Sammons, M.C. The Longman Guide to Style and Writing on the Internet, 2nd ed. Longman, 2007. ISBN: 020557629X. Parson, J. J., and Oja, D. New Perspectives on Computer Concepts: Comprehensive. 15th ed. Boston, Mass.: Thomson/Course Technology, 2012. ISBN 1133190561.

62

THE ORAL ASSESSMENT


Following the FSOT, the files (including the registration application form, personal narrative questionnaire, and FSOT scores) of candidates who passed the FSOT will be reviewed by a Qualifications Evaluation Panel. Those candidates selected will be invited to participate in the Oral Assessment, a series of exercises constituting the next stage of their candidacy. The principal Foreign Service Assessment Center is located in Washington, D.C. Centers may also be established for limited periods in other major U.S. cities. Oral Assessments will begin within several weeks after candidates have been notified of their eligibility, and they must be scheduled during a fourweek Oral Assessment window. Exceptions will be made on a case-by-case basis, but candidates must take the Oral Assessment within 4 months of the date they took the FSOT. Candidates must report to their assigned Assessment Center at 7:00 AM on their scheduled day. The assessment may end as late as 6:00 PM for successful candidates. Candidates who are invited to the Oral Assessment must complete and bring to the Oral Assessment a Statement of Interest, and they will be asked to complete an on-line security background form (SF86) to submit electronically in the event of passing the Oral Assessment. Candidates who ultimately receive a conditional offer of employment will be given additional required documents during the preemployment phase of their candidacy. Candidates with disabilities should notify the Assessment Center of any special needs in writing with supporting documentation when they return the confirmation form. To confirm these arrangements, candidates should also contact the Department of State at reasonableaccommodations@state.gov. The Oral Assessment is an examination, not a job interview. Oral Assessment exercises are based on a job analysis of the work of the Foreign Service and the skills, abilities, and personal qualities deemed essential to the performance of that work. The assessment measures the following 13 dimensions: Composure. To stay calm, poised, and effective in stressful or difficult situations; to think on one's feet, adjusting quickly to changing situations; to maintain self-control. Cultural Adaptability. To work and communicate effectively and harmoniously with persons of other cultures, value systems, political beliefs, and economic circumstances; to recognize and respect differences in new and different cultural environments. Experience and Motivation. To demonstrate knowledge, skills, or other attributes gained from previous experience of relevance to the Foreign Service; to articulate appropriate motivation for joining the Foreign Service. Information Integration and Analysis. To absorb and retain complex information drawn from a variety of sources; to draw reasoned conclusions from analysis and synthesis of available information; to evaluate the importance, reliability, and usefulness of information; to remember details of a meeting or event without the benefit of notes. Initiative and Leadership. To recognize and assume responsibility for work that needs to be done; to persist in the completion of a task; to influence significantly a groups activity, direction, or opinion; to motivate others to participate in the activity one is leading. Judgment. To discern what is appropriate, practical, and realistic in a given situation; to weigh relative merits of competing demands. Objectivity and Integrity. To be fair and honest; to avoid deceit, favoritism, and discrimination; to present issues frankly and fully, without injecting subjective bias; to work without letting personal bias prejudice actions. Oral Communication. To speak fluently in a concise, grammatically correct, organized, precise, and persuasive manner; to convey nuances of meaning accurately; to use appropriate styles of communication to fit the audience and purpose.

63

Planning and Organizing. To prioritize and order tasks effectively, to employ a systematic approach to achieving objectives, to make appropriate use of limited resources. Quantitative Analysis. To identify, compile, analyze, and draw correct conclusions from pertinent data; to recognize patterns or trends in numerical data; to perform simple mathematical operations. Resourcefulness. To formulate creative alternatives or solutions to resolve problems, to show flexibility in response to unanticipated circumstances. Working With Others. To interact in a constructive, cooperative, and harmonious manner; to work effectively as a team player; to establish positive relationships and gain the confidence of others; to use humor as appropriate.

Written Communication. To write concise, well organized, grammatically correct, effective, and persuasive English in a limited amount of time. Candidates are evaluated against these criteria by a team of examiners who observe the performance of candidates in a variety of situations designed to enable them to demonstrate the requisite skills. As trained observers with years of actual Foreign Service experience, the examiners are able to evaluate candidates and provide professional judgments of their performance. The Oral Assessment is not an adversarial process. Candidates do not compete against one another, but instead are judged on their capacity to demonstrate the skills and abilities sought by the Foreign Service.

For more information


Candidates who wish to read more about the Oral Assessment process or see practice scenarios from the Oral Assessment may visit the State Department website at the following address: careers.state.gov

64

Foreign Service Officer Test Study Guide, 5th Edition

*04507013ET*

Anda mungkin juga menyukai